MSS

1. A patient presented to a traumatologist with a trauma of shoulder. What wall of axillary cavity contains foramen trilaterum and foramen quadrilaterum? a) anterior b) posterior c) lateral d) medial e) intermediate 2. A patient presented to a traumatologist with a trauma of leg, which he had sustained at a sport competition. Upon examination, damage of posterior muscle, that is attached to calcaneus by its tendon, was found. This muscle is: a) triceps surae b) tibialis posterior c) popliteus d) fibularis longus e) fibularis brevis 3. In the course of a cesarean section, an incision was made in the pubic area and vagina of rectus abdominis muscle was cut. What does anterior wall of the vagina of rectus abdominis muscle consist of? A. aponeurosis of m. transversus abdominis, m. obliquus internus abdominis. B. aponeurosis of m. transversus abdominis, m. pyramidalis. C. aponeurosis of m. obliquus internus abdominis, m. obliquus externus abdominis. D. aponeurosis of m. transversus abdominis, m. obliquus externus abdominis. E. aponeurosis of m. transversus abdominis, m. obliquus internus abdominis 4. A 30 year-old woman complained of pain in the lower part of her forearm. Traumatologist found that her radio-carpal joint was damaged. This joint is: A. complex, ellipsoid B.simple, ellipsoid C.complex, cylindrical D.simple, cylindrical E.complex condylar 5. A woman was brought by an ambulance to the emergency department with a trauma of the cervical part of her vertebral column. Radiologist diagnosed a fracture of a nonbifid spinous processes of one of her cervical vertebrae. Spinous process of what cervical vertebra is fractured? A.VI. B. VII. C. III. D. IV. E. V. 6. A 5 year-old child, who was diagnosed with severe diphtheria, had been admitted to the infectious department. To prevent suffocation, the child had a tracheostomy done. In what triangle of the was the surgery performed? A.Omotracheale B.Caroticum C.Omoclaviculare D.Submandibulare E.Omotrapezoideum 7. A man was admitted to hospital with a fracture of the upper third of humerus with fragments displacement. The fracture was complicated by damage to the nerves and blood vessels that pass through foramen quadrilaterum that is located in the posterior wall of the axillary cavity. What structures limit foramen quadrilaterum? A. M. teres major, m. latissimus dorsi, m. biceps brachii, m. brachialis. B. M. teres major, m teres minor, m. triceps brachii, m. brachialis. C. M. latissimus dorsi, m. infraspinatus, m. coracobrachialis, m. brachialis. D. M. teres major, m. teres minor, m. triceps brachii, os humeri. E. M. teres major, m. biceps brachii, os humeri, m. brachialis. 8. A surgeon must amputate a gangrenous part of a foot at the level of Chopart joint. Which of the following ligaments is key for this surgery? a) lig.laterale b) lig.mediale (deltoideum) c) lig.bifurcatum d) lig.plantare longum e) lig.collaterale 9. A patient has sustained a trauma to his hip region that resulted in a fracture of os ilium in the area of spina iliaca anterior inferior. Some fibers of a ligament of the hip joint were also damaged. Fibres of which ligament were damaged? A. Lig. capitis femoris. B. Zona orbicularis. C. Lig.transversum acetabuli. D. Lig. iliofemorale. E. Lig. ischiofemorale. 10. A man is seen in a clinic for a deep cut to his hand in the area of intercarpal space. It was confirmed that his m. adductor pollicis had been damaged. What function is impaired in this case? A. Adductio B. Flexio C. Abductio D. Extensio E.Rotatio 11. A patient sustained a fracture of the upper part of humerus in a fall. One of his posterior muscles is dysfunctional, this muscle is attached to crista tuberculi minoris ossis humeri. What muscle is damaged? A. M. trapezius. B. M. rhomboideus major. C. M. rhomboideus minor. D. M. latissimus dorsi. E. M. serratus posterior superior. 12. A patient feels pain while breathing, especially on expiration. What muscle depresses ribs and takes part in expiration? A. M latissimus dorsi. B. M. serratus posterior inferior. C. M. rhomboideus major. D. M. trapezius. E. M. serratus posterior superior. 13. Upon examination of a 25 year-old patient a hematoma of soft tissues was found in the area of medial wall of axillary fossa. What chest muscle forms medial wall of the axillary fossa? A. M. pectoralis minor. B. M. pectoralis major. C. M. serratus posterior superior. D. M. serratus posterior inferior. E. M. serratus anterior 14. A 40 year-old patient felt pain after a fall. The doctor found a broken rib, which doesn’t have articular surface on tuberculum costae. What rib was damaged? A. V. B. XII. C. VII. D. X. E. III. 15. A patient’s mobility is limited after an awkward direct jump on his feet. The patient feels instability in the knee joint. What structures of the knee joint were damaged? A. medial meniscus B. lateral meniscus C. anterior crucial ligament D. posterior crucial ligament E. ligament of the head of a femur 16. A patient has sustained an intra-articular fracture of the neck of femur. Asymmetry of the head of femur is observed. What structure is damaged? A. Ligamentum capitis femoris B. Zona orbicullaris C. Nervus femoralis D. Nervus obturatorius E. Arteria obturatoria 17. During a final game a basketball player injured his right leg, after this injury he was unable to flex his right foot. The team’s doctor confirmed that a tendon was damaged. The tendon of what muscle was damaged? A. Musculus triceps surae (Achilles' tendon) B. Musculus extensor policis longus C. Musculus tibialis anterior D. Musculus biceps femoris E. Musculus gracilis 18. A man injured his femoral nerve while working; the nerve runs in lacuna musculorum. What structures limit lacuna musculorum? A. Lig. inquinale, lig lacunare, os pubis. B. Lig. inquinale, lig. lacunare, lig. Pectineale C. Lig. inquinale, os ilium, arcus ileopectineus. D. Lig. inquinale, os ilium, lig. pectineale. E. Lig. inquinale, arcus ileopectineus, os pubis. 19. Doctor suggested stimulating a reflex zone located in the area of an unpaired notch of the sternum in case of breathlessness during asthma attacks. What sternal notch is an unpaired one? A. Incisura costalis I. B. Incisura costalis II. C. Incisura jugularis. D. Incisura clavicularis. E. Incisura costalis VII. 20. A 5 year-old child suffers from neck deformation. Upon examination it is found that the head is tilted to the left, the face is turned to the right, passive motion of the head to the right is limited. What muscle is shortened? a) trapezius b) longus colli c) splenius capitis d) sternocleidomastoideus e) splenius cervicis 21. As a result of a fracture of tibia, anterior group of muscles of the lower leg was damaged. The function of what muscle may be affected? A. M.extensor hallucis longus. B. M.flexor digitorum longus. C. M. fibularis longus. D. M. soleus. E. M.extensor digitorum brevis 22. Upon examination of a patient it was noted that his facial expressions were somewhat limited: he is unable to pucker his lips, can’t whistle, the corners of his mouth do not rise up while smiling and the oral cleft is stretched sideways (transverse smile). The patient suffers from myopathy - degenerative hereditary disease with dystrophic muscle lesions. Atrophy of what muscle causes these symptoms? A. Musculus orbicularis oris B. Musculus zygomaticus major C. Platizma D. Musculus risorius E. Musculus masseter 23. A patient sustained a fracture of the upper part of humerus. One of the back muscle is dysfunctional, this muscle is attached to crista tuberculi minoris ossis humeri. What muscle is damaged? A. M. trapezius. B. M. rhomboideus major. C. M. rhomboideus minor. D. M. latissimus dorsi. E. M. serratus posterior superior. 24. A patient is diagnosed with oblique inguinal hernia, that came out of the inguinal canal. What structure forms inferior wall of inguinal canal? A. Ligamentum lacunare. B. Ligamentum inguinale. C. Fascia transversalis. D. M. transversus abdominis. E. Aponeurosis of m. obliqui externus abdominis. 25. In a dental practice a patient is seen with signs of impairment of saliva outflow from one of his parotid glands. Through what muscle does parotid duct run? A.M. masseter B.M. orbicularis oris C.M. zygomaticus major D.M. zygomaticus minor E.M. buccinator 26. As a result of an accident a patient feels pain and swelling in the anterior ankle area, supination of his foot is impossible and dorsiflexion is limited. The function of what leg muscle is most likely affected? A. M. flexor digitorum longus. B. M. tibialis anterior. C. M. flexor hallucis longus D. M. fibularis longus. E. M. fibularis brevis 27. In a young man, humerus is broken in the area of crista tuberculi majoris. Movements are limited. What muscle is attached to cristae tuberculi majoris humeri? A. M. pectoralis major B. M. pectoralis minor. C. M. serratus anterior. D. M. subclavius. E. M. transversus thoracis. 28. A sportsman has sustained a trauma of a joint of his lower limb and was admitted to the hospital following the accident. What is articulatio tarsi transversa composed of? a) art.talonavicularis and art.talocruralis b) art.calcaneocuboidea and art.subtalaris c) art.calcaneocuboidea and art.talonavicularis d) art.talonavicularis and art.cuneonavicularis e) art.talonavicularis and art.talonavicularis 29. A patient was admitted to the hospital with pain in a joint . Cruciate ligaments are located in: a) elbow joint b) knee joint c) hip joint d) shoulder joint e) wrist joint 30. During an operation (cesarean section), an incision was done in the pubic area and vagina of rectus abdominis muscle was cut. What does the anterior wall of vagina of rectus abdominis muscle consist of? A. Aponeurosis m. transversi abdominis, m. obliquus externus and internus abdominis. B. Aponeurosis m. transversi abdominis, m. pyramidalis.C. Aponeurosis m. obliqui internus abdominis, m. obliquus externus abdominis. D. Aponeurosis m. transversi abdominis, m. obliquus externus abdominis. E. Aponeurosis m. transversi abdominis, m. obliquus internus abdominis 31. A newborn has a congenital defect: mandibular median split. What process failed to fuse? A. Mandibularis B. Maxillaris C. Palatinus D. Frontalis E. Zygomaticus 32. A man went to the hospital with a brain injury. The doctor diagnosed a fracture of the skull. The line of fracture passes through linea nuchae superior. What bone was injured? A.os frontale B. os occipitale C. os parietale D. os temporale E. os ethmoidale 33. After an injury a boy has a fracture of patella. Function of what muscle is damaged? A. M. sartorius. B. M. biceps femoris. C. M. quadriceps femoris. D. M. semitendinosus. E. M. semimembranosus. 34. At a soccer match, a player suffered a chest trauma. Synchondroses are a type of: a) fibrous articulations b) synovial articulations c) cartilaginous articulations d) sutures e) gomphosis 35. A 30 year-old man went to the dentist complaining of chewing disorders, he suffers from pain in the jaw during retraction. The doctor found inflammation of one of the chewing muscles. What musle was it? A. Musculus temporalis (posterior part) B. Musculus temporalis (anterior part) C. Medial pterygium D. Lateral pterygium E. Masetter 36. In a case of pelvic injury the patient’s X-ray revealed necrosis of the head of femur. What ligament of the hip joint was damaged? A. Ligamentum capitis femoris B. Ligamentum ilio-femoralis C. Ligamentum pubo-femoralis D. Ligamentum ischio- femoralis E. Zona orbicullaris 37. A girl with long-standing history of nasal mucosa inflammation has signs of sphenoid sinus inflammation. What was the rout of infection spread? A. Recessus sphenoethmoidalis. B. Foramen sphenopalatinum C. Foramen ovale D. Foramen sinus sphenoidalis E. Foramen rotundum 38. A patient needs catheterization of subclavian vein. The catheter is placed in the area of trigonum clavipectorale. What structures limit it? A. Clavicula and superior border of musculus pectoralis minor. B. Clavicula and inferior border of musculus pectoralis minor. C. Clavicula and superior border of musculus pectoralis major. D. Inferior and superior borders of musculus pectoralis minor. E. Inferior border of musculus pectoralis major and inferior border of musculus pectoralis minor 39. An athlete felt pain along his sciatic nerve due to compression of the muscle that passes through incisura ischiadiaca major. What muscle was injured during exercise? A. Musculus piriformis B. Musculus gluteus medius C. Musculus obturatorius internus D. Musculus obturatorius externus E. Musculus quadratus lumborum 40. A victim injured his face and the outer surface of temporal region, this injury caused a fracture of zygomatic arch. What processes of skull bones were broken? A. Temporal process of the zygomatic bone and zygomatic process of the temporal bone B. Temporal process of the frontal bone and zygomatic process of the temporal bone C. Temporal process of the zygomatic bone and zygomatic process of the frontal bone D. Zygomatic process of the maxilla and zygomatic process of the temporal bone E. Zygomatic process of the maxilla and zygomatic process of the frontal bone 41. A 35 year-old man sustained a cut wound in the area of the palmar surface his thumb while working. Abduction of the thumb is now limited. Function of what muscle is damaged? A. opponens pollicis. B. palmaris brevis. C. abductor pollicis brevis. D. flexor pollicis brevis. E. adductor pollicis. 42. As a result of a fracture of tibia, the anterior group of muscles of the lower leg was damaged. The function of what muscle may be affected? A. M.extensor hallucis longus. B. M.flexor digitorum longus. C. M. fibularis longus. D. M. soleus. E. M.extensor digitorum brevis 43. A purulent process had spread from fossa canina to the soft tissues of orbit. Through what anatomical structure did the infection get into orbit? A. Canalis nasolacrimalis B. Foramen zygomaticoorbitale C. Foramen zygomaticofaciale D. Canalis infraorbitale E. Foramen zygomaticotemporale 44. As a result of an accident a patient has pain and swelling in the anterior ankle area, supination of the foot is impossible and dorsiflexion is limited. Function of what leg muscle is most likely affected? A. M. flexor digitorum longus. B. M. tibialis anterior. C. M. flexor hallucis longus D. M. fibularis longus. E. M. fibularis brevis 45. A 40 year-old man was taken to hospital with a wound of the anterior lower third of his shoulder. Flexion of shoulder and elbow joints is limited. Damage of what muscle is present? A. anconeus. B. brachialis. C. coracobrachialis. D. triceps brachii. E. biceps brachii 46. A patient presented to traumatologist with a trauma of that he had sustained during a sport competition. What type of joints is capable of movements in two planes? a) hinge b) pivot c) ellipsoidal d) cup-shaped e) gliding 47. A man sustained a cut wound of the lateral margin of the palmar surface of hand. The doctor discovered that thumb abduction was limited. Function of what muscle was damaged? A. Musculus abductor policis brevis B. Musculus opponens C. Musculus abductor policis D. Musculus palmaris brevis E. Musculi lubricales 48. Following an injury, a heavy bleeding from carotid artery branches is present. For temporary bleeding control, carotid artery should be pressed against the tubercle of the transverse processes of one of cervical vertebraes. What vertebrae is it? A. VI B. V C. IV D. III E. II 49. A surgeon during a surgery in a case of hard palate tumor, removed the tumor with a part of the upper jaw, which takes part in formation of the bony palate. What part of the bone was removed? A. Corpus B. Processus frontalis C. Processus zygomaticus D. Processus palatinus E. Processus alveollaris 50. A man was taken to an emergency room with a deep cut in the area of the I intercarpal space. Damage to m. adductor pollicis was confirmed. What function is impaired? A. Adductio B. Flexio C. Abductio D. Extensio E. Rotatio 51. Humerus is broken in the area of crista tuberculi majoris in a young male patient. Movements are limited. What muscle is attached to crista tuberculi majoris humeri? A. pectoralis major B. pectoralis minor. C. serratus anterior. D. subclavius. E. transversus thoracis. 52. A baby is seen in the emergency room with a wound of the right foot. During wound revision, the doctor noted a defect of a muscle tendon in close proximity to os cuboideum. The patient has limited ability to elevate the lateral side of the foot. Function of what muscle is impaired? A. triceps surae. B. tibialis anterior. C. extensor digitorum longus. D. quadriceps femoris. E. peroneus 53. Upon examination of a 2 year-old boy, an ophthalmologist noted impairment of tears outflow to meatus nasi inferior. What structure joins orbit with nasal cavity? A. Nasao- lacrimal canal. B. Anterior ethmoid foramen. C. Superior orbital fissure. D. Inferior orbital fissure. Optic canal. 54. An ambulance brought an 8 months old baby with an injury of the head in the anterior fontanel region. Damage of bone that limits this fontanel in the front was confirmed on X-ray. What bone was damaged? A. Os frontale B. Maxilla C. Os zygomaticus D. Os lacrimale E. Os palatinum 55. During examination of a 6 month old child a doctor noted that the posterior fontanel is not closed. At what age should it be closed in case of normal development of the child? A. Up to 3 months B. Up to 1 months C. Up to 6 months D. Up to 12 months E. Up to 24 months 56. A patient was admitted to the hospital with pain in the area of knee joint. A canal that leads from thigh to is called: a) cruropopliteal b) adductor c) musculoperoneus superior d) musculoperoneus inferior e) lateral 57. Following an injury, a boy has a fracture of patella. Function of what muscle will be impaired? A. sartorius. B. biceps femoris. C. quadriceps femoris. D. semitendinosus. E. semimembranosus. 58. While taking medical history of a patient, a doctor noted that a lesion was located in the . Which of the following anatomical structures is a border of this triangle? A. inferior border of the mandible B. midline of the neck C. m. sternocleidomastoideus D. m. trapezius E. m.omohyoideus 59. A patient presented to traumatologist with a trauma of arm that he had sustained during a sport competition. What type of joints is capable of movements in two planes? a) saddle b) pivot c) ellipsoidal d) plane e) ball-and-socket 60. A patient has superior displacement of acromial end of the clavicle. What ligaments are torn? a) anterior sternoclavicular, posterior sternoclavicular b) costoslavicular, interclavicular c) coracoclavicular, acromioclavicular d) acromioclavicular, costoclavicular e) costoslavicular, acromioclavicular 61. A patient was admitted to the hospital with shoulder joint pain. When in anatomical position, the subscapular fossa faces: a) anteriorly b) posteriorly c) medially d) laterally 62. During childbirth the head of a baby stopped at the entrance into the lesser . What structures form the boundary line that divides greater and lesser pelvis? A. Linea arcuata, sulcus limitans, promontorium. B. Linea arcuata, promontorium, margo inferius symphysis pubis. C. Linea arcuata, promontorium, margo superius symphysis pubis. D. Linea arcuata, symphysis pubis, basis ossis sacri. E. Cornu sacrale, linea arcuata. 63. A 45 year-old woman was taken to hospital with purulent inflammation of the tympanic cavity. Inflammation of nasal cavity and throat preceded infection spread to the tympanic cavity through the auditory tube. What formation of the base of the skull was the rout of the infection spread? A. Canalis caroticus B. Canaliculus tympanicus C. Canalis musculo-tubaris D. Canaliculus chordae tympani E. Canalis facialis 64. A patient was admitted to the hospital with acute pain in the gluteal area. A muscle that divides foramen ischiadicum majus into upper and lower parts is: a) quadratus femoris b) piriformis c) obturatorius externus d) obturatorius internus e)gemelli 65. A patient (35 year-old male) with a trauma of presented to emergency room. Cervical vertebrae can be distinguished by the presence of: a) transverse foramina b) round vertebral foramen c) fovea d) auricular surfaces e)bifida process 66. A patient was diagnosed with an abscess of the neck (purulent inflammation), that is located above manubrium sterni. What interfascial space the surgeon must open and drain? A. Spatium pretracheale B. Spatium previscerale C. Spatium interaponeuroticum suprasternale D. Spatium retropharingeale 67. A patient can’t adduct his right scapula to the vertebral column. What muscle is impaired? A. M. latissimus dorsi. B. M. rhomboideus major. C. M. levator scapulae. D. M. serratus posterior superior. E. M. serratus posterior inferior. 68. A patient can’t adduct his right scapula to the vertebral column. What muscle is impaired? A. M. latissimus dorsi. B. M. rhomboideus major. C. M. levator scapulae. D. M. serratus posterior superior. E. M. serratus posterior inferior. 69. In a patient a stabbed wound of the anterior chest wall (closer to the sternum) is present. Superficial muscles of the chest and membrana intercostalis externa are damaged. This membrane is direct continuation of: A. Mm. intercostales interni B. Mm. levatores costarum. C. Mm. intercostales externi. D. Mm. transversi thoracis. E. Mm. subcostales. 70. A young man during active pulling exercise experienced sharp pain in the back. Objectively: pain at the attempts of motion of upper extremity; adduction and pronation of arm are limited. Strain of what muscle most likely happened? a) rhomboideus major b) trapezius c) latissimus dorsi d) levator scapulae e) erector spine 71. In a patient a stabbed wound of the anterior chest wall (closer to the sternum) is present. Superficial muscles of the chest and membrana intercostalis externa are damaged. This membrane is direct continuation of: A. Mm. intercostales interni B. Mm. levatores costarum. C. Mm. intercostales externi. D. Mm. transversi thoracis. E. Mm. subcostales. 72. A man sustained an injury of the femoral nerve that runs in lacuna musculorum. What structures limit lacuna musculorum? A. Lig. inquinale, lig lacunare, os pubis. B. Lig. inquinale, lig. lacunare, lig. Pectineale C. Lig. inquinale, os ilium, arcus ileopectineus. D. Lig. inquinale, os ilium, lig. pectineale. E. Lig. inquinale, arcus ileopectineus, os pubis. 73. During physical exercise, a 15 year old student suddenly developed pain in the area of the hip joint during inward rotation and abduction of his lower limb. A doctor confirmed damage of one of the pelvic muscle, its tendon leaving the pelvic cavity through foramen ischiadicus major. What muscle was damaged? A. M. obturatorius internus. B. M. piriformis. C. M. gluteus medius. D. M. obturatorius externus. E. M. quadratus femoris. 74. is frequently used for injections of medicines and venous blood draws because of convinient ulnar vein location just under the skin. What structures limit cubital fossa? ?A. M. brachialis, m. brachioradialis, m. pronator teres. B. M. biceps brachii, m. brachioradialis, m. pronator teres. C. M. biceps brachii, m. brachioradialis, m. flexor carpi ulnaris. D. M. biceps brachii, m. brachioradialis, m. flexor carpi radialis. E. M. brachioradialis, m. pronator teres, m. flexor carpi ulnaris. 75. A 50 year-old man was taken to hospital with acute abdominal pain. The patient needs emergent surgery, in order to perform it the abdominal cavity must be widely open. Where should the surgeon perform the incision in order to avoid massive blood loss? A. On the lateral border of the rectus abdominis on the right site B. Through the rectus abdominis C. Through the linea alba D. Above the inguinal ligament on the right site. E. Above the inguinal ligament on the left site. 76. A sportsman sustained a trauma of a joint in the forearm region and was admitted to the hospital. Membrana interossea is a: a) suture b)synchondrosis c) symphysis d) syndesmosis e) gomphosis 77. A patient was admitted to the hospital with pain in a joint. What bone would be involved in case of a fracture along the intertrochanteric line? a) ilium b) femur c) tibia d) fibula e) patella 78. During an accident, the driver sustained numerous head injuries, including a fracture of the zygomatic arch. Function of what muscle, that is attached to the zygomatic arch, will be impaired? A. masseter. B. orbicularis C. buccinator. D. procerus. E. risorius 79. A patient complains of limited movements of the mandible on the right side. Function of what muscle is impaired? A. M. zygomaticus major B. M. temporalis dexter C. M. pterygoideus lateralis dexter D. M. pterygoideus lateralis sinister E. M. masseter sinister 80. In a 37 year-old patient has a foreign object in his airways, he suffers from cough and difficulty breathing. A tracheotomy was done on the neck, in the area that is limited by venter superior m. omohyoidei, m. sternocleidomastoideus and median line of the neck. In what triangle of the neck was the operation done? A. Trigonum caroticum. B. Trigonum omotracheale. C. Trigonum submandibulare. D. Trigonum omotrapezoideum. E. Trigonum omoclaviculare. 81. Following an injury, a 34 year-old patient is suffering from limited flexion of middle phalanges of fingers 2-5. Function of what muscles is most likely impaired? A. m. flexor digitorum superficialis B. m. flexor digitorum profundus C. m. opponens policis, m. adductor policis D. m. palmaris brevis, m. abductor digiti minimi E. m. palmaris longus 82. A patient (2year-old toddler) was admitted to the hospital with neurological symptoms and headache. What is the biggest fontanel? a) anterior b) posterior c) anterolateral d) posterolateral e) dorsolateral 83. At a soccer match, a player sustained trauma of a joint. Which of the following statements related to ligamentum capitis femoris is false? a) it is an intracapsular ligament b) it contains an arterial vessel supplying the head of femur c) it extends from fossa acetabuli to fovea capitis femoris d) it supports the hip joint e) it is an extracapsular ligament 84. A 35 year-old man with a trauma of his left palm was presented to a clinic. Upon examination the following was found: stabbed wound of left palm, middle phalanges of 2-5 fingers cannot flex. What structures were injured? a) lumbricales muscles b) tendons of deep flexor digitorum c) palmar interosseal muscles d) tendons of superficial flexor digitorum 85. At a soccer match, a player sustained a trauma of the knee joint. On X-ray a fracture of a bone located within the tendon of quadriceps muscle was confirmed. What group does this bone belong to? a) flat b) sesamoid c) tubular d) irregular e) round 86. A patient needs catheterization and injection of medicine into subclavian vein. The catheter is placed in the area of trigonum clavipectorale. What structures limit it? A. Clavicula and superior border of musculus pectoralis minor. B. Clavicula and inferior border of musculus pectoralis minor. C. Clavicula and superior border of musculus pectoralis major. D. Inferior and superior borders of musculus pectoralis minor. E. Inferior border of musculus pectoralis major and inferior border of musculus pectoralis minor 87. A 40 year-old woman presented to neurologist with complaints of pain and limitation of movements in the lumbar region. Radiologic findings revealed damage to sacroiliac lumbar connection. What processes of lumbar vertebrae are most likely damaged? A. Superior articular processes B. Inferior articular processes C. Superior and inferior articular processes D. Superior articular and transverse processes E. Inferior articular and transverse processes 88. During an operation in the area of the oral cavity floor a surgeon needs to locate the Pirogov’s triangle. What structures limit its upper border? A.N. hypoglossus B.M. stylohyoideus C.M. geniohyoideus D.M. digastricus E.M. mylohyoideus 89. A footballer visited his doctor complaining of sudden pain in the right knee during agame. Examination revealed displacement of the lateral meniscus. Which of the following structures is most likely to be damaged? A. Transverse ligament B. Anterior cruciate ligament and oblique ligament C. Cruciate ligaments and ligament of the patella D. Posterior cruciate ligament and oblique ligament E. Cruciate ligament and collateral ligaments 90. A man came to the hospital with a trauma of the head. Upon examination, the doctor confirmed a fracture of a facial bone of the skull. The fracture line passes through the condilar process. What bone was injured? A. Maxilla B. Mandibula C. Os zygomaticus D. Os lacrimale E. Os palatinum 91. A patient was diagnosed with dislocation of the acromial end of clavicle. A gap between ligaments of the acromiao-clavicular joint was found. What ligaments were damaged? A. Ligg.collaterale tibiale et fibulare B .Ligg.collaterale ulnare et radiale C. Ligg.sacrotuberale et sacrospinale D. Ligg.cruciatum anterius et posterius E. Ligg.coracoclaviculare et acromioclaviculare 92. A 57 year-old man with an injury of the head presented to an emergency room. Upon examination, the doctor found that the crack runs along the inferior surface of the temporal bone. What structure is located on the inferior surface? A. foramen jugulare B. foramen spinosum C. foramen caroticum internum D. hiatus canalis nervi petrosi minoris E. sulcus a. occipitalis 93. A 54 year-old man with an eyeball injury went to the hospital. Beside the damage to the eyeball, the doctor found a fracture of the orbital surface, orbital process and one of the parts of sphenoid bone. What part of the sphenoid bone was injured? A. Pterigoid process B. Ala minor C. Ala major D. Body E. Turkish saddle 94. A patient was diagnosed with brain damage due to a blow to his head. Median atlantoaxial joint is classified as: a) hinge b) gliding c) pivot d) condyloid e) ball-and- socket 95. A patient has purulent inflammation of the nasal cavity. The inflammation spread to the anterior cranial fossa. Through what anatomic structure did the inflammation spread? A. Foramina cribrosa. B. Foramen ovale. C. Foramen ethmoidalae posterius. D. Foramen sphenopalatinum. E. Foramen rotundum 96. A patient had trepanation and curettage of temporal bone cells (due to) purulent inflammation that spread from the middle ear. What bony process was the access site for the surgery? A. Processus mastoideus B. Processus zygomaticus C. Processus styloideus D. Processus pterygoideus E. Processus jugularis 97. A patient was hospitalized for correction of nasal septum deformation. What bones comprise the nasal septum? A. Perpendicular plate of the ethmoid bone and vomer B. Perpendicular plate of the palatine bone and vomer C. Perpendicular plate of the ethmoid bone and inferior nasal concha D. Nasal bone and vertical plate of the palatine bone E. Horizontal plate of the palatine bone and vomer 98. A patient sustained a fracture of the base of the skull. The line of fracture passes through foramen spinosum and foramen ovale. What cranial bone is injured? a) temporal bone b) occipital bone c) sphenoid bone d) palatine e) etmoid bone 99. On an x-ray image of pelvis of a newborn three distinct bones are seen, that are separated by cartilage in the area of the acetabulum. These bones are: a) os ilii, os sacrum, os coccygis b) os ilii, os pubis, os ischii c) os pubis os ischii, femur d) os sacrum, os pubis, os coccygis e) os pubis, os femur, os sacrum 100. A patient came to the hospital with a complaint of pain and limited movement of the radio-carpal joint. What movements are possible in the joint? A. Flexio, extensio, rotatio. B. Flexio, extensio, abductio, adductio, circumductio. C. Flexio, extensio, circumductio. D. Abductio, adductio, rotatio. E. Abductio, adductio, circumductio 101. A patient with a lacerated wound of the anterior lower third of forearm is seen in a hospital. He is unable to flex his wrist and proximal II-V phalanges. What muscle is damaged? A. flexor digitorum superficialis. B. flexor carpi ulnaris. C. flexor carpi radialis. D. brachioradialis. E. pronator quadratus. 102. A patient presented with a stabbed wound of the trapezius muscle. What fascia of the neck forms vagina for this muscle? A. Superficial cervical fascia B. Superficial layer of proper cervical fascia C. Deep layer of proper cervical fascia D. Prevertebral cervical the fascia E. Carotid vagina of cervical fascia 103. Cubital fossa is frequently used for injections and venous blood draws because of convenient ulnar vein location directly under the skin. What structures limit cubital fossa? A. M. brachialis, m. brachioradialis, m. pronator teres. B. M. biceps brachii, m. brachioradialis, m. pronator teres. C. M. biceps brachii, m. brachioradialis, m. flexor carpi ulnaris. D. M. biceps brachii, m. brachioradialis, m. flexor carpi radialis. E. M. brachioradialis, m. pronator teres, m. flexor carpi ulnaris. 104. A male patient sustained a fracture of the upper third of humerus with displacement of fragments, as a result blood vessels and nerves, that pass through foramen quadrilaterum of the posterior wall of axillary cavity, were damaged. What structures limit foramen quadrilaterum? A. M. teres major, m. latissimus dorsi, m. biceps brachii, m. brachialis. B. M. teres major, m teres minor, m. triceps brachii, m. brachialis. C. M. latissimus dorsi, m. infraspinatus, m. coracobrachialis, m. brachialis. D. M. teres major, m. teres minor, m. triceps brachii, os humeri. E. M. teres major, m. biceps brachii, os humeri, m. brachialis. 105. In case of an appendectomy surgeon dissects muscular fibers of anterior abdominal wall; chose the correct sequence: a) rectus abdominis, external oblique, internal oblique b) transversus, internal oblique c) external oblique, internal oblique, rectus abdomini d) external oblique, internal oblique, transversus e) rectus abdominis, external oblique 106. After an accident, a female patient was hospitalized with a fracture of temporal bone accompanied by bleeding from a sinus of the dura mater. Gap of the sinus is seen in the place where it is adjacent to a groove on the inner surface of the mastoid process. In what part of the temporal bone is damaged in this case? A. Sulcus sinus sagitalis superioris B. Sulcus sinus pertosi inferioris C. Sulcus sinus sigmoidei D. Sulcus sinus petrosi superioris E. Sulcus sinus transversi 107. During examination of a 25 year-old patient, a hematoma of the soft tissues of the medial wall of axillary fossa was found. What muscle of the chest forms the medial wall of axillary fossa? A. M. pectoralis minor. B. M. pectoralis major. C. M. serratus posterior superior. D. M. serratus posterior inferior. E. M. serratus anterior 108. Following a chest trauma a patient appears in pain during breathing, especially on expiration. What muscle depresses ribs and takes part in expiration? A. M latissimus dorsi. B. M. serratus posterior inferior. C. M. rhomboideus major. D. M. trapezius. E. M. serratus posterior superior. 109. A woman has an injury of radio-carpal joint with torn ligaments and damage to articular surfaces. What is the shape of radio-carpal joint? A. Art. elipsoidea. B. Art. condylaris. C. Art. plana. D. Art. cotylica 110. As a result of a fall, a 10 year-old child sustained a dislocation of sternum in the area of sterno-clavicular joint. What muscle strengthens this joint? A. M. pectoralis minor B. M. serratus anterior C. M. subclavius D. M. pectoralis major E. M. transversus thoracis 111. After an injury of occipital area, a crack in the region of transverse sinus was found. What part of the occipital bone is damaged? A. Squama B. Left lateral C. Right lateral D. Base E. Cerebellum 112. After the fall a woman sustained a fracture of olecranon of ulna, as a result she can’t extend her upper limb in the elbow and shoulder joints. Function of what muscle is impaired? A. M. biceps brachii. B. M. triceps brachii. C. M. anconeus. D. M. coracobrachialis. E. M. brachialis. 113. An 18 year-old patient had been suffering from sore throat went to the doctor after deterioration of his condition. Examination revealed a retropharyngeal abscess. In what space of the neck should the surgeon access the abscess? A. Retrovisceral B. Previsceral C. Inrtacervical D. Suprasternal E. Pretracheal 114. A patient has a penetrating wound in the center of his cheek, that requires stitching. What muscle will the surgeon stitch? A. M. buccinator B. M. masseter C. M. zygomaticus major D. M. orbicularis oris E. M. depressor anguli oris 115. A 62 year-old man has a lower back injury. An X-ray revealed a fracture of a processes of the III lumbar vertebra; the process is located in the sagittal plane. What processes are located in the saggital plane? A. Spinous and costal processes. B. Mastoid and spinous processes. C. Spinous and inferior articular processes. D. Superior articular and mastoid process. E. Superior, inferior articular and spinous processes. 116. A baby is seen in a clinic with a laceration of the right foot. During revision of the wound a doctor found a defect in a tendon that is projected onto os cuboideum. The patient’s ability to elevate the lateral foot side is limited. Function of what muscle is impaired? A. M. triceps surae. B. M. tibialis anterior. C. M. extensor digitorum longus. D. M quadriceps femoris. E. M. peroneus 117. A patient cannot close his mouth due to lower jaw drop. What muscles don`t execute their function properly? a) muscles of facial expression b) muscles of mastication c) orbicularis oris d) levator anguli oris e) depressor anguli oris 118. A 26 years old man has a work-related injury of the upper third of his shoulder. An examination didn’t show any damage of the bones. Active extension of the forearm is notably limited. What muscle is damaged? A. M. triceps brachii. B. M. coracobrachialis. C. M. biceps brachii. D. M. anconeus. E. M. pronator teres. 119. After a fracture of distal parts of both forearm bones treated with long-term immobilization a patient complains of limitated mobility of his wrist joint. What movements, characteristic for this particular joint, may be restored to full range by persistent training? a) flexion-extension, abduction-adduction b) flexion-extension, abduction-adduction, rotation c) rotation, flexion-extension d) flexion-extension e) abduction-adduction 120. A 26 years old man has a work-ralated injury of the upper third of his shoulder. An examination didn’t show any damage of the bones. Active extension of the forearm is notably limited. What muscle is damaged? A. M. triceps brachii. B. M. coracobrachialis. C. M. biceps brachii. D. M. anconeus. E. M. pronator teres. 121. A child fractured his humerus. The broken arm demonstrated decreased growth (compared to his other arm). What part of bone was affected? A. metaphysis B. epiphysis C. diaphysis D. apophysis E. medullary canal 122. A patient was admitted to a hospital with acute pain in her the forearm. What muscle provides movement in two joints? a) brachioradialis b) brachialis c) biceps brachii d) anconeus e) coracobrachialis 123. A patient was hospitalized due to a penetrating wound of the floor of his mouth. What muscle most likely is injured? A. M. mylohyoideus B. M. stylohyoideus C. M. geniohyoideus D. M. digastricus E. N. hypoglossus 124. During physical exercise a 15 year-old student suddenly developed pain in the area of his hip joint while rotating his lower limb inward. What muscle is damaged? A. Musculus gluteus medius B. Musculus piriformis C. Musculus obturatorius internus D. Musculus obturatorius externus E. Musculus quadratus lumborum 125. During examination of a patient, the doctor noted asymmetry of his face. The patient can not wrinkle his forehead, raise his eyebrows, whistle or close his eyes completely. Function of what muscles is impaired? A. Expression muscles B. Mustication muscles C. Infrahyoid muscles D. Subcutaneous muscles E. Suprahyoid muscles 126. A male patient is seen in an emergency room for a lacerated wound of the upper lateral third of the forearm. What muscle was damaged if the patient can not flex his upper limb in the elbow joint? A. M. extensor digiti minimi. B. M. extensor carpi radialis. C. M. extensor carpi ulnaris. D. M. brachioradialis. E. M. extensor digitorum. 127. An X-ray image of a patient showed destruction and enlargment of the Turkish saddle because of pituitary tumor. What bone structure was destroyed by the tumor? A. Sinus of sphenoid bone B. Canalis caroticus C. Canalis opticus D. Tympanic cavity E. Canalis facialis 128. A patient was admitted to the hospital with acute pain in a joint of his forearm. Which of the following is not characteristic for most synovial joints? a) articular cartilage b) synovial fluid c) meniscus d) joint capsule e) intracapsular ligament 129. A patient was diagnosed with purulent maxillary sinusitis. Pus from the maxillary sinus drains into: a) middle nasal meatus b) lower nasal meatus c) upper nasal meatus d) vestibule e) lateral nasal meatus 130. In an accident a driver sustained numerous head injuries, including fracture of the zygomatic arch. Function of what muscle, that is attached to the zygomatic arch, will be impaired? A. M. masseter. B. M. orbicularis C. M. buccinator. D. M. procerus. E. M. risorius 131. A 35 year-old man sustained a work-related cut wound in the area of palmar surface his thumb. Abduction of the thumb is limited. Function of what muscle is impaired? A. M. opponens pollicis. B. M. palmaris brevis. C. M. abductor pollicis brevis. D. M. flexor pollicis brevis. E. M. adductor pollicis. 132. A 45 year-old man went to a trauma center after a shoulder injury. Examination revealed absence of extension, adduction and pronation functions. Damage of what muscle could cause this situation? A. Musculus teres major B. Musculus subscapularis C. Musculus teres minor D. Musculus infraspinatus E. Musculus supraspinatus 133. A patient has pus accumulated within infratemporal fossa. Which of the following openings might be the rout of pus spread into the orbit? a) superior orbital fissure b) inferior orbital fissure c) infraorbital foramen d) foramen lacerum e) supraorbitalforamen 134. A male patient is seen in an emergency room for a lacerated wound of the upper lateral third of the forearm. What muscle was damaged if the patient can not flex his upper limb in the elbow joint? A. M. extensor digiti minimi. B. M. extensor carpi radialis. C. M. extensor carpi ulnaris. D. M. brachioradialis. E. M. extensor digitorum. 135. After the fall a woman sustained a fracture of olecranon of ulna, as a result she can’t extend her upper limb in the elbow and shoulder joints. Function of what muscle is impaired? A. M. biceps brachii. B. M. triceps brachii. C. M. anconeus. D. M. coracobrachialis. E. M. brachialis. 136. A patient suffered from an acute inflammation of the nasolacrimal canal. It is known that following the flu, the patient had long-standing history of nasal discharge. From what nasal cavity meatus could the infection spread to the nasolacrimal canal? A. From meatus nasi inferior B. From meatus nasi superior C. From meatus nasi media D. From meatus nasi communis E. From foramen sphenopalatinum 137. An oblique inguinal hernia that came out of the inguinal canal was diagnosed in a patient. What structure forms the inferior wall of the inguinal canal? A. Ligamentum lacunare. B. Ligamentum inguinale. C. Fascia transversalis. D. M. transversus abdominis. E. Aponeurosis of m. obliqui externus abdominis. 138. A patient with a lacerated wound of the lower anterior third of the forearm is sen in a clinic. The patient can neither flex his wrist, nor proximal phalanges of his II-V fingers. What muscle is damaged? A. M. flexor digitorum superficialis. B. M. flexor carpi ulnaris. C. M. flexor carpi radialis. D. M. brachioradialis. E. M. pronator quadratus. 139. During physical exercise, a 15 years old student suddenly felt pain in the area of his hip joint while rotating his lower limb inward and abducting it. The doctor confirmed damage of one of the pelvic muscles, whose tendon leaves the pelvic cavity through foramen ischiadicus major. What muscle is damaged? A. M. obturatorius internus. B. M. pinformis. C. M. gluteus medius. D. M. obturatorius externus. E. M. quadratus femoris. 140. A man was brought ot a surgical department with a wound on the medial side of the forearm. Examination revealed damage of a bony process, located next to incisura trochlearis . What bony process was damaged? A. Olecranon B. Procesus coronoideus. C. Processus styloideus D. Margo interosseus E. Tuberositas ulnae 141. A patient was admitted to the hospital with a disorder of facial expression. Facial nerve passes through foramen: a) ovale b) spinosum c) stylomastoid d) lacerum e) rotundum 142. A 30 year-old woman sustained an injury of sacrum in a fall. Radiologic imaging revealed damage of the sacral crest that is formed by transverse processes of the sacral vertebrae. What sacral crest was damaged? A. Middle and right medial crests. B. Right lateral and left lateral crests. C. Right lateral and right medial crests. D. Left lateral and left medial crests. E. Median and left medial crests. 143. A 27 year-old woman has a trauma of the lumbar region of her vertebral column. An X-ray image revealed a fracture of mastoid process of the II lumbar vertebra. Where are mastoid processes of the II lumbar vertebrae situated a? A. On the costal processes. B. On the spinous processes. C. On the inferior articular processes. D. On the superior articular process. E. On the superior and inferior articular processes. 144. A patient sustained a shoulder injury in the area of the large tubercle of humerus. He is unable to rotate his shoulder outside. What muscles are injured? A. m.infraspinatus et m.teres minor B. m. supraspinatus eti m. teres major C. m.subscapularis et m. coracobrachialis D. m.deltoideus et m. supraspinatus E. m. teres major et m. teres minor 145. An X-ray image showed intraarticular fracture of the proximal epiphysis of humerus. What structure of humerus is damaged? A. Caput humeri B. Collum chirurgicum C. Crista tuberculi minor D. Crista tuberculi major E. Tuberculum minor 146. Following an injury of the hip joint apatient has limitated mobility in it . What is the shape of the hip joint? A. Art. trochoidea. B. Art. ginglymus. C. Art. sellaris. D. Art. spheroidea. E. Art. cotylica. 147. A patient sustained a trauma to the medial aspect of the middle third of his leg due to blunt force impact. What anatomic structure is most likely fractured? a) diaphysis of tibia b) distal epiphysis of fibula c) proximal epiphysis of fibula d) distal epiphysis of tibia e) proximal epiphysis of tibia e) proxial epiphysis of femur 148. A surgeon is treating a deep wound on the outer surface of the right knee. What ligament must the doctor stitch? A. Lig. collaterale fibulare B. Lig. collaterale tibiale C. Lig. popliteum obliquum D. Lig. popliteum arcuatum E. Lig. patellae 149. While examining a person after a sport trauma, the physician noted a fracture of malleolus medialis. Tendons of what muscles might be injured? a) triceps surae, plantaris b) fibularis longus, fibularis brevis c) tibialis posterior, flexor digitorum longus, flexor hallucis longus d) stibialis anterior, extensor digitorum longus, extensor hallucis longus e) fibularis brevis, extensor digitorum longus 150. It is known, that fractures are common in the area of the surgical neck both in young and older patients. Indicate, on which of the following bones this landmark is present. a) humerus b) radius c) fibula d) femur e) scapula 151. An attack victim has a stabbed wound and muscles of the anterior wall of his axillary cavity are injured. Which muscles form this wall? A. M. pectoralis minor, m. subclavius B. M. pectoralis minor, m. pectoralis major. C. M. serratus anterior, m. subclavius D. M. serratus anterior, m. pectoralis major E. M. serratus anterior, m. pectoralis minor 152. To establish the boundary between the cervical and thoracic parts of vertebral column, the doctor must palpate and find the process of VIIth cervical vertebra. What process should the doctor be looking for exactly? A. spinosus B. transversus C. articularis superior D. articularis inferior E. mastoideus 153. In a patient a purulent process thet developed behind the may spread to: a) anterior mediastinum b) suprasternal space c) retromandibular fossa d) posterior mediastinum e) medial mediastinum 154. A patient is admitted with a brain injury. The doctor confirmed a fracture of a bone of the skull. The fracture line passes through protuberantia mentalis. What bone was injured? A. Mandibula B. Maxilla C. Os zygomaticus D. Os lacrimale E. Os palatinum 155. A 25 year-old patient complains of pain in the lumbar region. A defect of fusion of arches of 3rd and 4th lumbar vertebrae was found on x-ray examination. Name this developmental malformation: a) scoliosis b) spina bifida c) lordosis d) lumbalisation e) sacralisation 156. While examining a student who suffered a football injury, the physician noted that the patient`s left tibia could be moved anteriorly with considerable freedom, especially when the knee was flexed. This is indicative of a tear in the: a) anterior cruciate ligament b) lateral collateral ligament c) medial collateral ligament d) posterior cruciate ligament e) mediana collateral ligament 157. A 40 year-old man was taken to hospital with a deep laceration of the lower third of the anterior shoulder area. He has limited flexion in his shoulder and elbow joints. Damage of what muscle led to these disorders? A. M. anconeus. B. M. brachialis. C. M. coracobrachialis. D. M. triceps brachii. E. M. biceps brachii 158. A patient is admitted with a brain injury. The doctor confirmed an injury of a bone that takes part in formation of the lateral walls of the right orbit. What bone was damaged? A. Right frontal bone. B. Right zygomatic bone. C. Right sphenoid bone. D. Right ethmoid bone. E. Right maxilla. 159. A patient (68 year-old male) was admitted to the hospital with a problem in the . Posterior wall of rectus sheath below linea arcuata is formed by: a) fascia transversalis b) aponeurosis of transverses muscle c) peritoneum d) aponeurosis of internal oblique muscle e) aponeurosis of external oblique muscle 160. A 37 year-old patient has a foreign body in his airways; he suffers from cough and difficulty breathing. Tracheotomy was performed in the area that is limited by venter superior m. omohyoidei, m. sternocleidomastoideus and median line of the neck. In what triangle of the neck was the surgery done? A. Trigonum caroticum. B. Trigonum omotracheale. C. Trigonum submandibulare. D. Trigonum omotrapezoideum. E. Trigonum omoclaviculare. 161. An ambulance brought a man with a brain injury to the hospital. The doctor confirmed a skull fracture. The fracture line passes through the sagittal suture. What bone was injured? A. os frontale B. os occipitale C. os ethmoidale D. os temporale E. os parietale 162. Upon examination of a 45 year-old female patient, the doctor found that musculus teres minor was damaged. What function of the shoulder joint is lost? A. Adduction B. Flexion C. Abduction D. Extension E. Pronation 163. In a patient, a diaphragmatic hernia was diagnosed. What are weak points in the diaphragm? a) sternocostal and lumbocostal triangles b) centrum tendineum c) sternolumbar triangles d) lumbosternal triangles e) sternocostal and sternolumbal triangles

1. B 2. A 3. C 4. A 5. B 6. A 7. D 8. C 9. D 10. A 11. D 12. B 13. E 14. B 15. A 16. A 17. A 18. C 19. C 20. D 21. A 22. A 23. D 24. B 25. E 26. B 27. A 28. C 29. B 30. A 31. A 32. B 33. C 34. C 35. A 36. A 37. A 38. A 39. A 40. A 41. C 42. A 43. D 44. B 45. E 46. C 47. A 48. A 49. D 50. A 51. A 52. C 53. A 54. A 55. A 56. B 57. C 58. A 59. B 60. C 61. A 62. A 63. C 64. B 65. A 66. C 67. B 68. B 69. C 70. C 71. C 72. C 73. B 74. A 75. С 76. D 77. B 78. A 79. D 80. B 81. A 82. A 83. D 84. D 85. B 86. A 87. A 88. A 89. A 90. B 91. E 92. E 93. B 94. C 95. A 96. A 97. A 98. C 99. B 100. B 101. A 102. A 103. A 104. D 105. D 106. C 107. E 108. B 109. A 110. C 111. A 112. B 113. A 114. A 115. E 116. C 117. B 118. A 119. A 120. A 121. A 122. C 123. A 124. B 125. A 126. D 127. A 128. C 129. A 130. A 131. C 132. A 133. B 134. D 135. B 136. A 137. A 138. A 139. B 140. B 141. C 142. B 143. D 144. A 145. A 146. D 147. A 148. A 149. C 150. A 151. B 152. A 153. D 154. A 155. B 156. A 157. E 158. B 159. A 160. B 161. E 162. A 163. A

Splanchnology

164. A patient suffers from urinary retention. What genital gland is involved in the pathologic process? a) vesicula seminalis b) testis c) prostata d) bulbourethral gland e) epididimis 165. After an injury a man has problems in genitalia region. Which of the coats of testis derives from peritoneum, secretes serous fluid, and, in case of inflammation, may result in hydrocele (hydrops of testis)? a) fascia spermatica externa b) tunica dartos c) tunica vaginalis d) fascia spermatica interna e) m. cremaster 166. A patient has sublingual papilla inflammation. What salivary glands will be involved in salivation impairment? A) parotid and palatine B) parotid and submandibular C) sublingual and submandibular D) sublingual and parotid E) sublingual and buccal 167. A 35 year-old male came to the doctor with complaints of pain and swelling of the right testicle. On examination, a tumor that requires surgery was found, it is necessary to dissect seven layers of scrotum to tunica albuginea. Which tunica will the last one to be dissected before tunica albuginea? A) cutis orchis B) tunica spermatica externa C) tunica dartos D) tunica spermatica interna E) tunica vaginalis testis 168. A 3 year-old child was admitted to hospital with a foreign body in the bronchi. What bronchus did the foreign body most likely get into? A) in the right main bronchus B) in the left main bronchus C) in the right segmental bronchus D) in the left segmental bronchus E) in the lobar bronchus 169. Some children have predominantly mouth breathing due to excessive proliferation of lymphoid tissue. Proliferation of what structure is it due to? A) tonsils B) lingual tonsil C) tubal tonsils D) pharyngeal tonsil E) lymph nodes 170. A patient with impaired respiratory function must undergo tracheostomy. One should remember that the isthmus of the thyroid gland is often located at the level of the following cartilaginous rings of trachea: A) II- IV B) I-II C) IV-V D) V-VI E) III-IV 171. A patient has seizures. Hypofunction of what endocrine gland should you suspect? A) pituitary B) parathyroid gland C) gonads D) pineal body E) adrenal glands 172. During extirpation (removal) of the lateral lobes of thyroid gland, great care must be taken to avoid removing parathyroid glands, that are located: a) in front of the lobes b) behind the lobes c) laterally to the lobes d) medially to the lobes e) between the lobes 173. An instrument introduced into the space between lig. hepatorenale and lig. hepatoduodenale enters: a) bursa hepatica b) bursa omentalis c) bursa pregastrica d) superior duodenojejunal recess e) inferior duodenojejunal recess 174. After ovulation the oocyte is carried by ciliae of the fallopian tube epithelium to the uterus successively through: a) infundibulum, ampulla, isthmus, pars uterina b) ampulla, infundibulum, isthmus, pars uterina c) isthmus, ampulla, infundibulum, pars uterina d) infundibulum, isthmus, ampulla, pars uterina e)infundibulum, pars uterine, ampulla 175. Upon examination, a surgeon confirmed injuries to the middle third of the left kidney. The integrity of what organ should be checked while taking into account syntopy of the left kidney? A) pancreas B) the C) D) E) liver 176. A doctor diagnosed inflammation of the in the narrowest part of the larynx cavity in a 3-year old child. In what part of the child's larynx did the doctor find inflammation? A) aditus laryngis B) glottis C) vestibulum laryngis D) cavitas infraglottica E) ventriculus laryngis. 177. During chromocystoscopy, a doctor must find urethral orifice. What anatomical structure can serve as a reference point for this manipulation? A) cervix vesicae B) sorpus vesicae C) apex vesicae D) urachus E) trigonum vesicae 178. After a tooth extraction a doctor saw three roots . What tooth was extracted? A) canine of the upper jaw B) premolar of the upper jaw C) molar of the upper jaw D) premolar of the lower jaw molar E) molar of the lower jaw 179. Laboratory study of a 56 year-old patient's blood showed increase in blood sugar. What endocrine gland is damaged? A) glandula pineale B) glandula supratenalis C) glandula thyroidea D) glandula parathyroidea E) pancreas 180. A 35-year old man was admitted to the surgical department with a purulent wound on his neck in front of the trachea (around previsceral space). Where can infection spread if the patient is not operated on immediately? A) to the chest cavity - anterior mediastinum B) to the chest cavity - posterior mediastinum C) to the retrovisceral space D) to the spatium interaponeuroticum suprasternale E) to the chest cavity - the middle mediastinum 181. A patient has thyroid dysfunction. The examination revealed decrease in the number of blood triiodothyronine and tetraiodothyronine. What is the structural and functional unit of thyroid gland? A) particles B) follicles C) slices D) stroma E) thyrocytes. 182. There is a need for bladder catheterization in a male patient. In which urethral part or structure can resistance occur? A) external urethral opening B) spongy part C) membranaceus part D) prostate E) internal urethral opening. 183. A patient has sustained a stabbed wound to the soft tissues of the face in the area of masseter muscle. What gland’s duct may be damaged? a) parotid b) lesser sublingual c) greater sublingual d) submandibular e)palatal 184. Chronic kidney disease was complicated be stone release from the kidney. At what level of the ureter is it most likely to stop? A) at the border of abdominal and pelvic parts B) in the place of renal pelvis transition into the ureter C) in the abdominal part D) 2 cm above the confluence of the bladder E) 5 cm above the pelvic part 185. A patient was admitted to the hospital with a problem in the pelvic region. The urinary bladder from the inside can be seen with a cystoscope that is introduced through urethra. At what part of urinary bladder may trigonum vesicae be identified? a) apex b) corpus c) fundus d) cervix e) radix. 186. A patient complains of food particles getting into his nasal cavity while swallowing. What muscle of the soft palate is affected? A) tensor veli palatine B) glossopharyngeal C) musculus uvulae D) palatoglossus E) palatopharyngeus, 187. A sportsmen has sustained a trauma of the abdominal region and was admitted to the hospital. All of the following are true with respect to the diaphragmatic surface of the liver, except: a) this area is traversed by inferior vena cava b) opening on the area is porta hepatic c) this area is circumscribed by coronary ligament d) this area is traversed by inferior vena cava e) this area is partly devoid of peritoneum 188. A woman presented to the dentist with complains of loss of taste on the top of her . The doctor found that, due to long-standing history of smoking, her taste buds were damaged. What papillae were damaged? A) conicae B) fungiformes C) foliatae D) filiformes E) vallatae 189. During gynecological intervention - a hysterectomy, a surgeon must tie uterine artery which runs between the leaves of the broad ligament of the uterus. What organ can be damaged during this manipulation? A) fallopian tubes B) bladder C) urethra D) E) ovary. 190. An otolaryngologist diagnosed inflammation in a 12 year-old girl’s upper respiratory tract which includes all of the following, except: A) oral part of the pharynx B) external nose C) nasal cavity D) laryngeal vestibule E) nasal part of the pharynx 191. Upon examination of a patient, a surgeon found hydrocele. Between the layers of what testicular membranes can fluid accumulate? A) tunica albuginea B) tunica dartos C) external spermatic D) serous (vaginal). E) inner spermatic. 192. Upon a child's oral cavity examination, a dentist found short frenulum of the upper lip. What group of teeth will this anomaly have negative impact on? A) medial upper incisors B) medial lower incisors C) lateral upper incisors D) lateral lower incisors E) The upper canines 193. A dentist destroys cement connections between tooth root and dental alveolus during tooth extraction. What is this structure? A) periodontium B) gingiva C) pulpa dentis D) dentinum E) cementum 194. A patient undergoes left pulmonectomy. What is the correct location of anatomical structures in the root of the left lung (from top down)? a) artery, veins, bronchus b) bronchus, artery, veins c) artery, bronchus, veins d) veins, artery, bronchus e) artery, bronchus, artery 195. A woman has a problem with her genitalia. Which of the following structures derives from ductus paramesonephricus? a) tuba uterina b) ductus deferens c) ductus epididymidis d) ductus excretorius e) ductus ejaculatorius 196. A patient complains of urinary retention in. Pathology of what gland can cause this condition? A) prostate B) epididymis C) testicle D) seminal vesicle E) glandula bulbourethralis. 197. A patient has a fracture of the external nose and needs surgery. What does the external nose consist of? A) wing, tip, back, root B) base, tip, ala, neck C) back, root, dorsum, body D) back, tip, dorsum, root E) root, base, nostrils, ala 198. A mother has come to the pediatrician with her 14 months-old baby who has fever, is crying, and puts toys into his mouth. What tooth erupts at this age? A) first upper molar _ B) medial lower incisor C) premolar D) сanine E) lower second molar _ 199. Patient B., aged 39, who cannot become pregnant for eight years, was advised to see an endocrinologist. An examination of the patient revealed exophthalmos, eyelid tremor, and tachycardia. What endocrine gland is affected? A) Thyroid B) Pancreatic C) Genital D) Adrenal E) Epiphysis 200. For surgical access to larynx, it is necessary to determine the cartilage that contains oblique line. What cartilage of larynx has oblique line? A) cricoid B) cuneiform C) thyroid D) corniculate E) arytenoid 201. A patient has taste impairment, while touch, pain and temperature sensations are intact. What papillae have no taste receptors? a) filiform b) vallate c) foliate d) emboliformis e) fungiform 202. In order to confirm a diagnosis, a 70-year old patient needs palpation of pelvic organs through the anterior wall of rectum. What organs can be examined this way? A) vagina, ovaries. B) uterus, vagina. C) ovaries, uterus. D) fallopian tubes, vagina. E) fallopian tubes, uterus. 203. A patient complains of urine leaking during sexual intercourse. What organ is affected? A) prostate. B) seminal vesicles. C) testicle. D) epididymis. E) bladder. 204. On examination of oral cavity, a dentist noted a child has large lower molar teeth. What is the child's age? A) 8-9 years B) 4-5 years C) 6-7 years D) 10-11 years E) 12-13 years 205. On examination of oral cavity, a child has been diagnosed with inability to stick the tongue out. What tongue muscle is damaged? A) Hyoglossus muscle. B) Genioglossus muscle. C) . D) Superior longitudinal muscle. E) Inferior longitudinal muscle. 206. A patient diagnosed with pyelonephritis was admitted to urology department. After examination, upwards spread of infection accompanied by reflux pyelovenosus was found. Lesion of what structure led to the complications? A) direct tubes. B) excretory kidney ducts. C) fornical apparatus of the kidney. D) renal tubules. E) renal corpuscles. 207. Inflammation of modified subserous layer around the cervix caused intense pain in a patient. Pathological process of what area was confirmed by the doctor? A) Endometrium B) Myometrium C) Mesometrium D) Parametrium E) Perimetrium 208. A 28 year-old patient was admitted to nephrology department with prolapse of the right kidney (nephroptosis). X-ray contrast urography confirmed the preliminary diagnosis. What relations to the 12th rib does the right kidney normally have? A) 12th rib crosses the kidney in the upper third. B) 12th rib crosses the kidney center. C) 12th rib is projected on the upper pole D) 12th rib crosses the kidney in the lower third. E) 12th rib is projected on the lower pole 209. A 28-year old woman was diagnosed with ectopic pregnancy that was complicated by fallopian tube rupture. In what peritoneal space will blood accumulate? A) intersigmoidal sinus B) vesicouterine C) right mesenteric sinus D) left mesenteric sinus E) rectouterine 210. A patient complains of headache following a rhinitis. After examination and an x- ray, maxillary sinusitis is confirmed. What way did infection spread into this sinus? a) posterior ethmoid cells b) ethmoid infundibulum c) anterior ethmoid cells d) hiatus maxillaris e) anterior ethmoid cells 211. A 10-year old patient was admitted to hospital after swallowing a peanut; he is coughing constantly an is in considerable distress. His voice is unchanged. Where is the foreign body most likely located? A) Trachea B) Right main bronchus C) Left main bronchus D) Rima vestibule E) Glottis 212. A 25 year-old patient complains of difficulty swallowing. A tumor is confirmed in the area of foramen caecum. What structure is foramen caecum a remnant of? a) sublingual b) thymus c) thyroid d) parathyroid e) submandibular 213. On examination of oral cavity a child is found to have lower first permanent molar teeth. How old is the child? A) 8-9 B) 4-5 C) 6-7 D) 10-11 E) 12-13 214. Examination of a 4-year old child revealed face puffiness, dilation of veins of the upper body, X-ray showed increased shadow of mediastinal organs. Defect of what immune organ led to clinical manifestations of the disease? A) rear mediastinal lymph nodes; B) thymus; C) sternal bone marrow; D) anterior mediastinal lymph nodes; E) diaphragmatic lymph nodes; 215. A 28 year-old patient is diagnosed with acute inflammation of nasolacrimal duct. From what part of nasal cavity could infection spread to the nasolacrimal duct? a) superior nasal meatus b) middle nasal meatus c) inferior nasal meatus d) vestibulum of nasal cavity e) common nasal meatus 216. In adults, calcium is frequently deposited in the epiphysis cerebri and then serves as a landmark on an x-ray examination of brain. Epiphysis cerebri (pineal body) belongs to: a) mesencephalon b) telencephalon c) myelencephalon d) metencephalon e) diencephalon 217. In a course of a gall-bladder surgery a doctor must define common bile duct. What anatomic structure is it in? a) gastrocolic ligament b) hepatorenal ligament c) hepatoduodenal ligament d) gastrorenal ligament e) hepatogastric ligament 218. No testicles were found in the right side of scrotum of a newborn boy. What kind of anomaly is this? A) Cryptorchidism B) Monorchism C) Hypospadias D) Hermaphroditism E) Epispadias 219. A 28-year old patient was admitted to hospital with acute abdominal pain. Upon examination the doctor diagnosed a liver and bile ducts pathology. Secretions from liver and pancreas are excreted into: a) superior part of b) descending part of duodenum c) horizontal part of duodenum d) ascending part of duodenum e) inferior part of duodenum 220. A child has chronic tonsillitis. His pediatrician recommends removal of the tonsils located in tonsillar fossa. What tonsils does the doctor plan to remove? A) Tonsilla pharyngealis. B) Tonsilla tubaria. C) Noduli lymphoidei solitarii. D) Tonsillae palatinae. E) Tonsilla lingualis. 221. A 13 year-old patient removed the second milk molar. What permanent tooth will grow in its place? A) second molar B) first premolar C) first molar D) second premolar E) third molar 222. A patient has notable redness of the mucous membrane of the tongue root. Determine what formation is involved in the inflammatory process. A) Palatine veil B) palatine tonsils C) tubal tonsils D) lingual tonsil E) pharyngeal tonsil 223. During appendectomy a doctor found some elements that belong to the peripheral organs of the immune system in the wall of the . What kind of elements are these? A) iliac nodes. B) single lymphoid nodules. C) clusters of lymphoid nodules. D) appendix nodes. E) paracolic lymphatic nodes. 224. A patient presented to nephrologist with complaints of intense pain in lumbar region and was diagnosed with acute kidney obstruction caused by a stone located at the border between abdominal and pelvic parts of the right ureter. What is anatomical boundary between these two parts? a) linea arcuata b) linea terminalis c) linea semilunaris d) linea alba e) linea transversa 225. A19 year-old male was diagnosed by ultrasound scan with kidney prolapse on routine pre-military service check-up. At the level of what vertebrae are kidneys usually located? A) 9-12 thoracic B) 4-5 lumbar C) 12 thoracic and 1 lumbar D) 9-10 thoracic E) 11 thoracic and 3 lumbar 226. A patient with cheek injury was admitted to hospital. What salivary gland duct may be damaged in this case? A) molar B) submandibular C) sublingual D) parotid E) buccal 227. A woman was diagnosed with ovarian cancer. She needs a surgery to remove the ovary. What ligament must be cut by surgeon to separate the ovary from uterus? a) lig.umbilicale mediale b) lig. teres uteri c) lig.ovarii proprium d) lig.suspensorium ovarii e) lig.umbilicale mediale 228. A 5 year-old had a tooth with two roots removed. What was thistooth? A) premolar of the upper jaw B) premolar of the lower jaw C) canine of the upper jaw D) molar of the lower jaw E) molar of the upper jaw 229. A patient was diagnosed with bartholinitis (inflammation of the greater vestibule glands). What genital organ are these glands located in? A) vagina B) labia minora C) clitoris D) labia majora E) uterus 230. Chronic rhinitis was complicated by inflammation of frontal sinus. What nasal meatus did the infection spread from to this sinus? a) superior b) middle c) common d) left e) inferior 231. On examination of the oral cavity, a doctor found swelling and redness of a structure located between the arches. Which anatomical formation is inflamed? A) Tonsilla tubaria B) Tonsilla palatine C) Tonsilla adenoidea D) Tonsilla pharyngea E) Tonsilla lingualis 232. A doctor examines a boy aged 16. The patient has underdevelopment of genitals, there is no hair on the pubis and torso, his muscles are underdeveloped due to delays in ossification; the patient is tall and has body disproportion. There is no spermatogenesis. In the blood, lower than normal levels of testosterone are detected. What cells produce this hormone? A) Interstitial endocrinocytes (Leydig cells). B) Parafollicular endocrinocytes (C-cells). C) endocrinocytes of zona fasculata. D) Endocrinocytes of the reticular zone. E) Insulocytes. 233. A patient came to the doctor with complaints of tremor of his fingers and body, muscle weakness, palpitations, sleep disorders, weight loss with increased appetite. Symptoms of what gland's disorder are observed in the patient? A) Pituitary B) Pancreatic C) Thyroid D) Adrenal E) Epiphysis 234. During gynecological examination a patient was diagnosed with endometritis (inflammation of the endometrium). What part of the uterine wall is affected by inflammation? A) parametrium B) C) muscular coat D) E) Mucosa 235. A child has orthodontic problems caused by persistent mouth breathing, as breathing through the nose is difficult. What tonsils are hypertrophied? A) palatine and tubal B) tubal C) lingual D) palatine E) pharyngeal 236. During examination of a patient doctor found hypertrophy and inflammation of lymphoid tissue, swelling of the mucous membrane between arches of the soft palate (acute tonsillitis). What tonsils are normally contained in this place? A) Tonsilla tubaria B) Tonsilla pharyngealis C) Tonsilla palatina D) Tonsilla lingualis E) Tonsilla adenoidea 237. A patient (68 year-old male) was admitted to hospital with a problem in chest region. Which of the following is not characteristic of the left lung? a) cardiac impression b) superior lobe c) inferior lobe d) middle lobe e) intermedial lobe 238. A dentist filled a cavity between two lower central incisors. On what surface of the crown did the dentist work? A) Facies mesialis B) Facies labialis C) Facies lingualis D) Facies distalis E) Facies occlusalis 239. During examination a doctor found hypertrophy and inflammation of the lymphoid tissue, edema of the mucousa between arches of the soft palate (acute tonsillitis). Which of the tonsils is normally located in that place? A) Tonsilla palatina B) Tonsilla pharyngealis C) Tonsilla tubaria D) Tonsilla lingualis E) Tonsilla nasalia 240. A woman was hospitalized with a diagnosis of acute adnexitis on the right. Folloing additional diagnostic work-up acute appendicitis was confirmed. What position does the appendix occupy in this case? a) descending b) ascending c) lateral d) retrocaecal e) medial 241. A patient has intense bleeding from the renal artery due to a stabbed wound in abdominal area. Where will hematoma be located in this case? A) retroperitoneal space B) peritoneal cavity C) abdominal cavity D) greater pelvic cavity E) pelvic cavity 242. What part of colon may be damaged in case of a stabbed wound in the left side of abdomen? a) colon ascendens b) colon descendens c) colon transversum d) rectum e) colon sigmoideum 243. A patient is suffering from hydrocele. The lesion of what testicular membrane can lead to this condition? A) Tunica vaginalis testis B) Fascia spermatica externa C) Fascia spermatica interna D) Fascia cremasterica E) Tunica dartos 244. A 28-year old patient was diagnosed with acute inflammation of mucous membrane of the naso-lacrimal duct. According to his history, after 10-day long flu, he had nasal discharge. From what part of the nasal cavity could infection spread to the naso-lacrimal duct? A) From the lower meatus B) From the upper meatus C) From the middle meatus D) From the vestibule of the nose E) From the frontal sinus 245. A patient was admitted to the hospital with acute headache. What is not a paranasal sinus? a) palatine sinus b) ethmoid sinus c) sphenoid sinus d) right maxillary sinus e)left maxillary sinus 246. In a patient during examination of the oral cavity a doctor found redness of mucosa in the area of tongue root, palatine veil, and palatoglossal arches. Limits of whast anatomical formation are inflamed? A) pharyngeal tonsil B) lingual tonsil C) fauces D) palatine tonsils E) tubal tonsils 247. A patient was admitted to the hospital with an abdominal problem. Tunica muscularis of what tubular organ of digestive canal consists of three layers? a) duodenum b) c) d) e) stomach 248. The doctor diagnosed a foreign body in the cavity of larynx in the area of elastic cone in a 4-year old child. Where is the foreign body logged? A) In laryngeal ventricle B) In the laryngeal vestibule. C) At the entrance to the larynx. D) In the infraglottic cavity. E) In the vocal cords. 249. In a patient, liquid food gets into the nasal cavity through the roof of the mouth due to damage of a bony formation. Which anatomic formation is damaged? A) soft palate. B) root of the tongue. C) circular muscle of mouth. D) pharyngeal opening of the auditory tube (eustachian cushion). E) hard palate. 250. A child has RDS (respiratory distress syndrome) due to lack of surfactant that covers: a) wall of alveola b) wall of trachea c) wall of bronchus d) wall of bronchiolus e) wall of pharynx 251. On examination of the oral cavity of a 5 year-old child the dentist found abnormality of tooth enamel development. The enamel develops from: A) mandibular bone B) entoderm of the main colon C) ectoderm of the oral cavity D) mesoderm E) mesenchyme 252. A 16-year old girl was diagnosed with immune organ dysfunction. An immune organ that also belongs to the endocrine system is: A) lymph nodes B) palatine tonsils C) spleen D) pituitary E) thymus 253. In patient had a tooth removed. The crown of the tooth has rhomboid shape with four tubercles on the chewing surface; the tooth has three roots. Determine what tooth was removed. A) lower second molar B) first lower molar C) upper second molar D) upper third molar E) upper first molar 254. A woman was diagnosed with ovarian tumors. She needs a surgery. What ligament should the surgeon cut to separate the uterus from the ovary? A) suspensory ligament B) lateral umbilical ligament C) broad ligament D) ovary ligament E) round ligament of the uterus 255. A patient has taste disturbance and cannot discern sweet and salty at the apex and around the edges of the tongue. What tongue papillae affected? A) Papillae filiformes B) Papilla vallatae C) Papilla conicae D) Papillae fungiformes E) Papillae foliatae 256. On examination of the oral cavity a dentist noticed a child had his first permanent canines present. What is the age of the child? A) 13 years . B) 10 years. C) 9 years. D) 7 years. E) 6 years. 257. To determine the type of leukemia, a patient has to undergo bone marrow study. Puncture of what bone should a doctor perform? A) head of the IIIrd rib B) diaphysis of humerus C) diaphysis of femur D) Ist cervical vertebra E) sternum 258. A man complains of a sore throat. Examination revealed hypertrophy of lymphoid organ, located in fossa tonsillaris. What is this organ? A) Tonsilla palatina. B) Tonsilla pharingea. C) Tonsilla tubaria. D) Tonsilla lingualis. E) Tonsilla adenoidea. 259. Following an examination of a newborn, organ development abnormality was found. What organ does not develop from the maxillary process? A) upper jaw B) cheeks and side parts of the upper lip C) hard and soft palate D) cheek bone E) lower jaw 260. On examination of the oral cavity decay of a tooth crown on the side facing the proper oral cavity was noted. How is this surface called? A) Facies vestibularis. B) Facies lingualis. C) Facies contactus. D) Facies mesialis. E) Facies distalis. 261. A patient’s nose bleeds from a. sphenopalatina, branches of which come in the nasal cavity through foramen sphenopalatinum. The doctor performs a deep nasal packing pushing gauze into the nasal part of the pharynx. What passage connects the nasal cavity with the throat? A) Meatus nasi superior. B) Meatus nasi medius. D) Meatus nasopharyngeus. C) Meatus nasi inferior. E) Meatus nasi communis. 262. Due to orchitis (inflammation of the testicles) a 43 year-old patient has problems with sperm production. In what part of the testicle did the pathological process occur? A) Ductus epididimidis B) Ductuli seminiferi recti C) Ductuli eferentes testis D) Ductuli seminiferi contorti E) Rete testis 263. In a 35 year-old patient a cyst was found behind median raphe the tongue in the region of the cecal foramen. Which embryonic structure is not reduced? A) lingual tonsils. B) thyroglossal duct. C) lingual papilla. D) cecal foramen. E) boundary raphe of tongue. 264. A patient has aspermatism. What organ is affected? A) prostate. B) epididymis. C) testicle. D) prostate gland. E) seminal vesicles. 265. A patient is undergoing left-sided pulmonectomy due to lung cancer. A surgeon should recall the order of anatomical structures of the right lung root (top down). A) veins, bronchus, artery B) artery, bronchus, veins C artery, vein, bronchus D) vein, artery, bronchus E) bronchus, artery, vein 266. In the cystoscope field flat mucosa with no folds is seen. What part of the bladder is in the field? A) top. B) cystic triangle. C) body. D) neck. E) bottom 267. A 10 year-old child had his first milk molar removed. What permanent tooth will grow in its place? A) second premolar B) first premolar C) first molar D) second molar E) third molar 268. A young woman was admitted to the hospital with complaints of sharp pain in the abdomen. On examination, a doctor suspects rupture of the fallopian tube due to ectopic pregnancy. During puncture through the posterior vaginal vault, blood was found in the syringe. In what peritoneal space does blood accumulate in this case? A) rectouterine recess. B) rectovesical recess. C) intersigmoidal recess. D) vesicouterine space. E) space behind the blind duct. 269. A patient has inflammation of the sphenoid sinus. Where does its aperture open? A) Meatus nasi communis B) Meatus nasi medius C) Meatus nasi inferior D) Recessus sphenoethmoidalis E) Infundibulum 270. During a difficult delivery pubic symphysis ruptured. What organ is most likely to be damaged? A) Rectum B) Bladder C) Ovaries D) Fallopian tubes E) Uterus 271. A child was diagnosed with congenital cleft upper lip and cleft palate. Failure of fusion of what processes caused this defect? A) maxillary, nasal B) nasal, palatal C) palatal, mandibular D) mandibular, nasal E) nasal, frontal 272. During a right-sided lobectomy, a surgeon approached the root of the right lung for the purpose of separation and processing of its components. Specify the order of the elements of the root of the right lung from top to bottom? A) Pulmonary artery, bronchus, pulmonary veins B) bronchus, pulmonary artery, pulmonary vein C) Pulmonary vein, pulmonary artery, bronchus D) Bronchus, pulmonary artery, phrenic nerve E) Phrenic nerve, bronchus, bronchial artery and vein 273. Upon examination of the oral cavity a dentist revealed inflammation of the tissues surrounding a tooth. What anatomical formation is affected? A) Paradontium B) Cementum C) Gingiva D) Alveola dentalis E) Periodont 274. A mother with a 2 year-old child came to pediatrician. How many baby teeth should this child have? A) 8 B) 18 C) 16 D) 10 E) 20 275. A newborn baby chokes on milk while breastfeeding due to milk getting into his nasal cavity. What kind of developmental abnormality is this? A) cleft lip. B) esophagus narrowing. C) anus atresia. D) cleft palate. E) esophageal atresia. 276. A patient with sinusitis has impaired connection between maxillary paranasal sinus and middle meatus. Which of the following usually ensures air flow from the nasal passage to the maxillary sinus? A) Hiatus semilunaris B) Foramina ethmoidalia C) Recessus sphenopalatinus D) Canalis nasolacrimalis E) Infundibulum ethmoidale 277. A 45-year old male was hospitalized with glomerulonephritis. The urinanalysis showed presence of blood cells. What part of the nephron is damaged? A) Renal corpuscle B) Proximal straight tubule C) Distal convoluted tubule D) Proximal convoluted tubule E) Distal straight tubule 278. On examination of the oral cavity a dentist noticed appearance of first permanent lateral incisors in a child. What is the age of the child? A) 9 B) 6 C) 7 D) 11 E) 13 279. After a tooth extraction a dentist saw that the tooth had two roots. What is the tooth? A) Canine B) Upper molar C) Lower molar D) Lower premolar E) Upper incisor 280. Patient aged 30 with inflammation of the pulp of his 2nd upper molar came to a doctor complaining of headaches and discharge from the nose. After examination he was diagnosed with pulpitis complicated by sinusitis. What sinuse did the infection spread to from the tooth root? A) sphenoid B) maxillary C) ethmoid D) frontal E) mastoid cells 281. A patient with abdominal injury was admitted to hospital. Tunica muscularis of all below mentioned tubular organs consists of two layers, except: a) esophagus b) duodenum c) stomach d) ileum e) jejunum 282. A 65 yea-old patient came to the hospital with complaints of difficulty urinating. Examination revealed hypertrophy of prostate. Increase of what prostate part is a possible cause of this condition? A) Left part B) Isthmus (medium part) C) Prostate capsule D) Right part E) Prostatic duct 283. A patient is diagnosed with acute necrotic pancreatitis. What peritoneal space will effusion spread to immediately? a) bursa omentalis b) bursa hepatica c) bursa pregastrica d) right lateral canal e) left lateral canal

284. A patient has Conn's syndrome - adenoma of the glomerular zone of adrenal cortex with increased secretion of hormones that leads to sustained hypertension. Which hormone is produced by glomerular zone of adrenal cortex? A) Androstenediol. B) Cortisol, corticosterone. C) Aldosterone. D) Androgens, estrogens, progesterone. E) Epinephrine, norepinephrine. 285. A doctor examining a child's nasal cavity, found mucosal proliferation in the region of infundibulum. In what paranasal sinus is there difficulty of air passage? A) Sinus sphenoidalis B) Sinus maxillaris. C) Anterior ethmoid bone cells. D) Middle ethmoid bone cells. E) Sinus frontalis. 286. A doctor checks nasolacrimal duct patency. Which passage of the nasal cavity does the duct open into? A) Upper nasal. B) General nasal. C) Middle nasal. D) Inferior nasal. E) Naso-pharyngeal. 287. A 22-months old child has 18 teeth. What teeth can be expected to cut in the forseeable future? A) mandibular canine teeth B) upper canine teeth C) 2 lower molars D) lateral lower incisors E) 2 upper molars in 288. A patient developed acute anuria (absence of urine) after hysterectomy. What anatomical structures were most likely damaged during the surgery? A) urethra. B) ureter C) external sphincter of the urethra. D) internal sphincter of the urethra. E) bulbospongiosus muscle. 289. A doctor examining nasal cavity of a woman, who is a heavy smoker, found inflammation of the mucous membrane in the area of canalis incisivus. Which nasal meatus does it open in? A) Meatus nasopharyngeus. B) Meatus nasi superior. C) Meatus nasi communis. D) Meatus nasi medius. E) Meatus nasi inferior. 290. A 58-year old man came to urologist complaining of sharp pain during urination and reduction in the amount of urine. The doctor suggested the presence of urinary stone disease. In what part of male urethra can stones most likely be stuck? A) Pars pelvina B) Pars spongiosa C) Pars membranacea D) Pars intramuralis E) Pars prostatica 291. A 3 year-old child was hospitalized with the foreign body in his bronchus. What bronchus did the foreign body most likely get into, if its diameter was approximately 1,5 cm? a) in right medial segmental bronchus b) in left main bronchus c) in right main bronchus d) in left lobar bronchus e) in left medial segmental bronchus 292. A patient with a tumor of the left main bronchus has complaints about difficulties in the passage of food. Which organ may be involved in the malignant process? A) Heart. B) The main bronchi. C) Retrosternal gland. D) Trachea. E) Esophagus. 293. On examination of oral cavity a dentist discovered carious cavity on the surface of the crown of a premolar, that borders the first molar. Name the damaged surface of the crown. A) Facies mesialis B) Facies distalis C) Facies vestibularis D) Facies lingualis E) Facies occlusalis 294. Mother of a 9-year old girl came to an endocrinologist complaining of breast enlargement, reddish vaginal discharge, increased hair growth on the body and around the vulva. What endocrine gland is affected (that was supposed to inhibit premature puberty)? A) Epiphysis cerebri B) Gl. Thyroidea C) Hypophysis cerebri D) Gl. Suprarenalis E) Gl. parathyroidea 295. On oral cavity examination of a 7 year-old child a dentist found a growing tooth behind the second milk molar. What is this tooth? A) third permanent molar B) second permanent molar C) first permanent molar D) first permanent premolar E) second permanent premolar 296. In case of a mid-line neck injury, a patient has intense bleeding and difficulty breathing. Examination showed that, in addition to injured trachea, thyroid isthmus was also damaged. What level in relation to trachea is the wound located at? A) II-III tracheal cartilage B) IV-V tracheal cartilage C) V-VI tracheal cartilage D) III-IV tracheal cartilage E) VI-VII tracheal cartilage 297. On examination of oral cavity a dentist discovered decay on the surface of the first lower premolar crown turned to the upper teeth. What surface is this? A) Facies occlusalis B) Facies vestibularis C) Facies ingualis D) Facies mesialis E) Facies distalis 298. A 10 days old child had a surgery to correct cleft lip that is the result of: A) nonunion of the third gill arch B) nonunion of palatal plate of the maxillary process of the 1st arch. C) nonunion of the second arch. D) nonunion of the medial nasal and maxillary process of the 1st arch. E) nonunion of the maxillary and mandibular processes of the first arch. 299. A dentist extracted a tooth with a big diamond-shaped crown, four tubercles and three roots (2 buccal and 1 lingual ). What tooth is this? A) Lower 1st molar B) Upper 1st molar C) Upper 2nd molar D) Lower 2nd molar E) Upper 3 molar 300. In children often difficulty in nasal breathing is common due to enlargement of pharyngeal lymphoid tissue. What tonsils are most likely the cause of this condition? a) lingual b) pharyngeal c) palatine d) mandibular e) tubal 301. A tooth was removed. The tooth has oval-shaped chewing surface, deep transverse groove separates buccal and lingual tubercles; the root is flattened in mesio- distal direction, has longitudinal grooves and a cleft. Determine what tooth was removed. A) first upper premolar B) lower canine C) first lower premolar D) second upper premolar E) upper canine 302. In a patient with an injury of the skull base hamulus of medial plate of sphenoid bone is damaged. What muscle of soft palate is torn? A) Palatopharyngeus B) Palatoglossus C) Levator veli palatini D) Musculus uvulae E) Tensor veli palatini 303. A patient has inflammation of the alveolar prosess of the upper jaw (osteomyelitis). What tooth structure can be primarily affected by this disease? A) Enamel B) Pulp C) Dentin D) Periodont E) Cement 304. A 50 year-old man was taken to hospital with acute abdominal pain after a stabbing. What part of colon may be damaged if the wound is located on the right side of the abdomen? a) colon ascendens b) colon descendens c) colon transversum d) colon sigmoideum e) rectum 305. Immunologist sees a teenage girl with impairment of B – lymphocytes production. Which hematopoietic and immune system organ is the source of B - lymphocytes? A) tonsils B) red bone marrow C) thymus D) spleen E) isolated lymph nodes 306. A patient has impairment of pain and temperature sensitivity of the tongue. What papillae are affected? A) foliate papillae, fungiform papillae B) filiform papillae, conical C) vallate papillae, filiform papillae D) conical, vallate papillae E) fungiform papillae, filiform papillae 307. 27-year old male is seen in a clinic with complaints of enlargement of hands, feet and lower jaw. Additionally deformities of joints and vertebral column as well as hormonal disorders (impotence, testicular atrophy) are present. What gland is affected? A) anterior pituitary B) adrenal glands C) pineal body D) thyroid gland E) parathyroid glands 308. A 12 months old baby is seen in a clinic. The baby has fever, is crying and putting toys into her mouth. What tooth erupts at this age? A) first lower molar B) medial lower incisor C) premolar D) Canine E) lower second molar 309. A 60 year-old patient after a surgery is in need of urinary catheter placement. In what part of urethra is there risk of potential trauma associated with this manipulation? a) pars prostatica b) ostium urethrae externum c) pars spongiosa d) pars cavernosa e) pars membranacea 310. A patient has obturative jaundice due to malignant tumor. During surgery a surgeon takes into account that papilla duodeni major is the site of opening of: a) hepato- pancreatic ampule b) ductus cysticus c) common hepatic duct d) left hepatic duct e) right hepatic duct 311. A patient has Recklinghausen's disease which occurs in case of parathyroid glands hyperfunction. The examination revealed kidney stones, bone distortion, missing teeth, spontaneous fractures that occur often . The increased amount of what hormone was detected in the patient’s plasma? A) triiodothyronine. B) parathyrin. C) calcitonin. D) tetraiodothyronine. E) thyrotropin. 312. On oral cavity examination, enlargement of sublingual papilla to the right of the tongue frenulum is noted. Specify glands that would have their excretory function. A) sublingual and parotid B) thyroid and parathyroid C) submandibular and sublingual D) parotid and thyroid E) parotid and submandibular 313. A patient was admitted to the hospital with an abdomianl problem. Double layer of peritoneum that supports the intestinal tract is called: a) visceral peritoneum b) mesentery c) greater omentum e) excavation d) lesser omentum 314. An 18-year old man was diagnosed with abnormal ejaculation in the output phase (accumulation of secretions in the prostate part of the male urethra). What duct is the pathway of sperm and seminal fluid to the male urethra? A) Ductus ejaculatorius. B) Ductus excretorius. C) Ductus deferens. D) Ductus epididymidis. E) Ductuli prostatici. 315. Which of following teeth are permanent and not deciduous? a) medial incisors b) canines c) premolars d) molars e) lateral incisors 316. A patient has a tumor in the inferior part of the bladder. What organ can it damage taking into account syntopy of the bladder? A) rectum. B) spermatic cord. C) prostate. D) ampulla of the deferent duct. E) seminal vesicles. 317. A dental examination revealed inflammation of the tissues surrounding a tooth. What tissues are inflamed? A) Paradont B) Gingiva C) Pulpa dentis D) Dentinum E) Cementum 318. A 4 year-old girl was diagnosed with shortened frenulum of the upper lip. What teeth can be affected by this anomaly? A) lateral lower incisors B) medial lower incisors C) lateral upper incisors D) medial upper incisors E) canines 319. A dentist found carious cavity on the surface of the second molar, that is facing the first molar. Identify the affected surface. A) Facies vestibularis B) Facies distalis C) Facies lingualis D) Facies mesialis E) Facies occusialis 320. A woman was taken to hospital with symptoms of acute abdomen. On examination, ectopic pregnancy was suspected. What anatomical structures of the pelvis should be punctured to confirm the correct diagnosis? A) Excavatio rectouterina B) Excavatio vesicouterina C) Excavatio rectovesicalis D) Fossa ischiorectalis E) Processus vaginalis peritonei 321. In a patient, pneumonia was complicated by exudative pleuritis. Where can fluid accumulate in such case? A) sinus obliquus pericardii B) sinus phrenicomediastinalis pleurae C) sinus transversus pericardii D) sinus costodiaphragmaticus pleurae E) sinus costomediastinalis pleurae 322. A dentist discovered carious cavity on the surface of the crown of a lower canine turned to the first premolar. Name this surface. A) Facies distalis B) Facies vestibularis C) Facies lingualis D) Facies mesialis E) Facies occlusalis 323. A 15 year-old patient needs sialography of parotid gland. Where does parotid duct open? A) In the sublingual region B) on the cheek in the area in front of the 2nd lower tooth C) on the cheek in the area in front of the upper 2nd premolar D) In caruncula sublingualis E) on the cheek in the area in front of the upper 2nd molar 324. During bladder catheterization, following rapid catheter insertion, bleeding due to injury of urethra mucous membrane in the area of voluntary sphincter appeared. What part of the urethra should be crossed with caution in order to avoid trauma? A) In pars membranacea uretrae B) In bulbus uretrae C) In pars spongiosa uretrae D) In the area of fossa navicularis uretrae E) In parts prostatica uretrae 325. On cystoscopy, under normal conditions, bladder mucosa forms folds except one triangular area where the mucosa is smooth. In what part of the bladder is this triangle situated? A) Bladder neck B) Top bladder C) Body of the bladder D) Isthmus of the bladder E) Bottom of the bladder 326. A patient was admitted to hospital with acute pain in the oral cavity. What muscle dilates glottis? a) cricothyroid b) posterior cricoarytenoid c) lateral cricoarytenoid d) vocalis e) anterior cricoarytenoid 327. Patient has difficulty discerning bitter taste. What tongue papillae are affected? A) Papillae conicae B) Papillae foliatae C) Papillae vallatae D) Papillae filiformes E) Papillae fungiformes 328. Examination of a 47 year-old woman revealed urine reflux. What part of kidney are the smooth muscles that prevent urine from flowing backwards located? A) Pelvis renalis B) Calyx renalis minor C) Ductuli papillares D) Capsula glomeruli E) Calyx renalis major 329. During a surgery for femoral hernia a surgeon operates within the boundaries of . What structure makes up its upper margin? A) Lig.lacunare. B) Fascia lata. C) Lig.pectinale. D) Arcus iliopectineus. E) Lig.inguinale. 330. A 35 years-old patient has been admitted to a surgical ward with a suppurative wound on the neck, anterior to trachea (previsceral space). Unless a surgery is performed urgently, there is a risk of infection spreading to: A) Thoracic cavity - posterior mediastinum. B) Retrovisceral space. C) Thoracic cavity-anterior mediastinum D) Interaponeuritic suprasternal space. E) Thoracic cavity-middle mediastinum.

164. c 165. c 166. c 167. e 168. a 169. d 170. a 171. b 172. b 173. b 174. a 175. a 176. b 177. e 178. c 179. e 180. a 181. b 182. c 183. a 184. a 185. c 186. a 187. c 188. b 189. c 190. a 191. d 192. a 193. a 194. c 195. a 196. a 197. a 198. a 199. a 200. c 201. a 202. b 203. a 204. c 205. b 206. c 207. d 208. c 209. e 210. d 211. b 212. c 213. c 214. b 215. c 216. e 217. c 218. b 219. b 220. d 221. d 222. d 223. c 224. b 225. e 226. d 227. c 228. d 229. d 230. b 231. b 232. a 233. c 234. e 235. e 236. c 237. d 238. a 239. a 240. a 241. a 242. b 243. a 244. a 245. a 246. c 247. e 248. d 249. e 250. a 251. c 252. e 253. e 254. d 255. d 256. a 257. e 258. a 259. e 260. b 261. c 262. d 263. b 264. c 265. e 266. b 267. b 268. a 269. d 270. b 271. a 272. b 273. a 274. e 275. d 276. a 277. a 278. a 279. c 280. b 281. c 282. b 283. a 284. c 285. c 286. d 287. c 288. b 289. e 290. c 291. c 292. e 293. a 294. a 295. c 296. a 297. a 298. d 299. b 300. b 301. a 302. e 303. d 304. a 305. b 306. b 307. a 308. a 309. a 310. a 311. b 312. c 313. b 314. a 315. c 316. c 317. a 318. d 319. d 320. a 321. d 322. a 323. e 324. a 325. e 326. b 327. c 328. b 329. e 330. c

CNS 331. A 55 year-old patient complained of gait disturbance and inability to maintain vertical position, thus was admitted to the hospital. He was diagnosed with significant white matter degeneration of the cerebellum, middle cerebellar peduncles and olivo- cerebellar pathways. Through which structures of the CNS does tractus olivocerebellar pass? a) Superior cerebellar peduncles b) Inferior cerebellar peduncles c) Midle cerebellar peduncles d) Pedunculicerebri e) Posterior crus of internal capsule 332. In a patient, a brain tumor was diagnosed, located in the posterior horn of the lateral ventricle and the area of the "calcarine spurs." Disturbance of which function will develop if the tumor continues to grow rapidly? a) Disturbance of visual function b) Disturbance of olfactory function c) Disturbance of taste d) Disturbance of vestibular analyzer e) Disturbance of acoustic function 333. A patient was admitted to a hospital with a hemorrhage that occurred in the area of "sulcus calcarinus". What function of the body was damaged? a) Somatosensory b) Olfactory c) Auditory d) Motor function e) Visual function 334. In a patient a loss of general sensation in some areas on one side of the body was observed after a trauma to the head. Which cerebral gyrus was damaged? a)upper temporal b)precentral c)postcentral d)lower temporal e)middle temporal 335. In a patient a lesion of anterior horns of the spinal cord by a tumor was revealed. What neurons are affected? a) Parasympathetic b) sensory c) Sympathetic d) Motor e) Mixed 336. A patient complains of losing the ability to distinguish colors. Upon examination a retinal structure responsible for color sensitivity was found to be damaged. What is this structure? a) Cons b) Rods c) Bipolar neurons d) Multipolar neurons e) Ganglionic neurons 337. A 49 year-old woman was diagnosed with Parkinson’s desease. An MRI image demonstrates degradation of substantia nigra. Substantia nigra is a source of: a) dopamine b) adrenaline c) acetylcholine d) serotonine e) melatonin 338. A 63 year-old patient, who works as a carpenter, presented to a neurologist with a complaint of inability to perform precise movements with his right hand for the past 3 months.The doctor found that the patient has damage of: a) Gyrus supramarginalis. b) Gyrus postcentralis. c) Gyrus precentralis. d) Gyrus temporalis superior. e) Gyrus angularis 339. A patient has trauma-ralated paralysis on the right side of the body and disturbance of pain sensitivity on both the right and left sides of his body. What is the reason of his condition? a) damage of spinal cord on the right side b) damage of the cerebellum c) damage of the motor zone of cerebral hemisphere d) damage of the midbrain e)damage of caudate nucleus 340. In a 34 year-old patient after meningoencephalitis, an acute decrease in hearing was noted. Examination of the patient has excluded pathology of the sensory and conductive hearing apparatus. Which gyrus of the cerebral cortex is involved? a) Superior temporal b) Superior frontal c) Supramarginal d) Angular e) Middle temporal 341. As a result of a road accident, a 29 year-old patient was taken to hospital with a trauma of the vertebral column. Examination revealed right-sided paralysis of the lower limb with increased muscle tone. What part of the central nervous system was damaged by the injury? a) Anterior horn of spinal cord b) Posterior horn of spinal cord c) Anterior funiculus of spinal cord d) Medulla e) Right cortico-spinal tract 342. Following cranial trauma, a patient has lost taste and smell. Which cortical center was damaged? a) uncus b) insula Reili c) lower frontal gyrus d) upper parietal lobule e) middle temporal gyrus 343. A patient with clinical dysfunction of the spinal cord at the level of lumbar segments needs surgery. Where are the segments located? a) Th XII - L I b) Th VII - X c) Th XI - XII d) Th Х - XІ e) L I - II 344. As a result of a road accident, a 35 year-old patient has been taken to the hospital with a spinal injury. Examination revealed right-sided paralysis of the lower limb with increased muscle tone due to damage of the cortico-spinal tract. Where is the crossing of the piramidal tract located? a) Capsula interna b) Midbrain c) Medulla d) Pons e) Thalamus 345. A 35 year-old patient was hospitalized with an injury to cervical segments of the spinal cord after a road accident. How many segments does this part of the spinal cord have? a) 7 b) 8 c) 5 d) 12 e) 2 346. A patient was admitted to the hospital with suspected cerebral bleeding. A CT scan is peformed and a bleed in the area of trigonum lemnisci was found. What pathway does this particular site belong to? a) Tr. Frontopontinus b) Tr.opticus c) Tr acousticus d) Tr. Tectospinalis e) Tr. Reticulospinalis 347. In a patient, a brain tumor in the area of the posterior crus of the internal capsule (the posterior part) was discovered on a CT image by a doctor. What functional disorders might be present in this patient? a) Vision and hearing b) Voluntary movement of the head and neck muscles c) Voluntary movement of the body and limbs d) Proprioceptive and exteroceptive sensation from the opposite side e) Olfaction 348. Patient's head injury caused loss of tactile and temperature sensations. Which gyrus was affected? a) Precentral b) Angular c) Supramarginal d) Postcentral e) Cinguli 349. A patient becomes tired quickly while working. In a standing position with his eyes closed he is unstable and loses balance. Skeletal muscle tone is decreased. Which of the brain structures is most likely injured in this man? a) Thalamus b) Basal ganglia c) Limbic system d) Precentral gyrus of hemispheres e) Cerebellum 350. After a brain injury a 47 year-old patient seeked medical help because he was unable to perform fine motor actions, such as buttoning up, lighting a match, pouring a glass of water. Examination has found that muscle strength, deep muscle sense and coordination mechanisms were preserved. Which area of the cerebral cortex was affected? a) Supramarginal gyrus b) Near the calcarine sulcus c) Precentral gyrus d) Transverse temporal gyrus e) angular gyrus 351. A 46 year-old patient with glaucoma has significant increase in intraocular pressure with hypersecretion of aqueous humour. Damage of which structure is the disturbance of liquid circulation connected with? a)sinus venosus sclerae b)iris c)choroids d)ciliary body e) ciliary muscle 352. Upon examination of a 54 year-old patient a tumor in pons, particularly in the area of corpus trapezoideum, was found. What pathway is this area a part of? a) Tr. Acusticus b) Tr. spinothalamicus lateralis c) Tr. spinocerebellaris anterior d) Tr. spinocerebellaris posterior e) Tr. spinothalamicus anterior 353. A 41 year-old patient was diagnosed with a bleeding in the posterior horn of the spinal cord. What is the function of the posterior horns? a) Mixed b) Motor c) Sympathetic d) Parasympathetic e) Sensory 354. A child was admitted to the hospital and diagnosed with a fracture of the base of his skull. Bulbar disorders were significant. In the area of what fossa did the fracture occur? a) temporal b) anterior c) middle d) posterior e) infratemporal 355. A patient has paralysis of his right upper and lower limbs. A center of which cerebral hemisphere is damaged? a) Center of simultaneous rotation of the head and eyes to the opposite side of the right hemisphere b) Motor center of left hemisphere c) Motor center of right hemisphere d) Center of precise movement of the left hemisphere e) Center of stereognosis of the left hemisphere 356. An 82 year-old patient complains of taste loss. What parts of the cortex are damaged? a) Uncus and hyppocampus b) Inferior frontal lobe and subcalose area c) Angulare gyrus and hyppocampus d) Uncus and inferior area of postcentral gyrus e) Subcalose area and cingulate gyrus 357. A 36 year-old patient has motor disfunction of the tongue after a viral infection. Pathology of what nerve should be suspected? a)vagus b)hypoglossal b)lingual d)glossopharyngeal e)facial 358. A 27 year-old woman came to an ophtalmologist complaining of blurred vision. Examination has found disturbances of lens accommodation process. Function of what structures is impaired in the patient? a) M. Ciliaris b) M. dilatator pupillae c) M. sphincter pupillae d) Lig. pectinatum iridis e) Corpus vitreum 359. A patient's brain computer tomography has showed a tumor that is localized in the area of inferior salivatory nucleus. What part of the brain is affected? a) Medulla b) Midbrain c) Diencephalon d) Pons e) Cerebellum 360. A patient after suffering a disorder of brain circulation has lost the ability to write letters and numbers. In which part of the brain did the pathology occurr? a) Lobus parietalis b) Lobus temporalis c) Lobus frontalis d) Lobus occipitalis e) Insula 361. A 65 year-old patient has impairment of motor centers that regulate the activity of the muscles of the head following a trauma. In which area of the cerebral cortex are the centers localized? a) Upper part of the precentral gyrus b) Lower part of the precentral gyrus c) Marginal gyrus d) Superior parietal lobe e) Angular gyrus 362. A 12 year-old boy was brought to a clinic by his parents. His height is 2 meters. On X-ray of his skull, enlargement of Turkish saddle was found. What brain structure has been affected in this case? a) Epiphysis b) Hypophysis c) Thalamus d) Midbrain e) Medulla 363. A patient can't understand text (can't read) because of complication after a trauma to the head. Where is the lesion localized? a) Superior temporal gyrus b) gyrus angularis c) Superior frontal gyrus d) Middle temporal gyrus e) Middle frontal gyrus 364. In a 49 year-old patient white matter of the spinal cord within the lateral part of the posterior cord (funiculus) is damaged; he has signs of proprioception disturbance in the upper limb. The fibers of which pathway form the lateral part of posterior cord (funiculus)? a) Tr. spinothalamicus lateralis b) fasciculus cuneatus tr.bulbothamici c) Tr. spinocerebellaris anterior d) Tr. spinocerebellaris posterior e) Fasciculus gracilis tr.bulbothamici 365. A 26 year-old patient has lost the ability to read and write after a surgery. What area of cortex was damaged? a) Gyrus supramarginalis b) Cuneus c) Sulcus calcarinus d) Gyrus parietalis superior e) Gyrus angularis 366. A patient, who sustained a trauma to the head 3 months ago, hears and understands speech, can talk, but cannot correctly name objects or recognize them. Which area of the cerebral cortex was damaged? a)lower frontal b)middle temporal c)upper frontal d)lower temporal e) occipital 367. A patient after a car crash 3 weeks ago complaints of disturbance of emptying the urinary bladder. A while later periodic involuntary emptying was stabilized. Injury of what part of CNS caused such condition? a)spinal cord (C6) b) frontal lobes c) nucleus caudatus d) sacral segments e) medulla 368. A subarachnoid posttraumatic hematoma was revealed in a patient's parietal region. Overall sensitivity loss was found by the doctor. What gyrus was subjected to compression? a) Postcentral b) Precentral c) Superior temporal d) Angular e) Superior parietal 369. A 67 year-old patient was admitted to the hospital after a hemorrhage in the area of thalamus. What receptors gather information that is transmitted to the cortex bypassing thalamus? a) touch b) vision c) hearing d) smell e) proprioceptive 370. A 62 year-old patient was admitted to the hospital with a hemorrhage in the area of inferior cerebral peduncles. What conducting pathway does not form the inferior cerebellar peduncle? a) tractus spinocerebellaris anterior b) tractus spinocerebellaris posterior c) fibrae arcuatae externae d) fibrae olivocerebellaris e) fibrae vestibulocerebellaris 371. After the brain injury, a patient has lost the ability to perform complex coordinated movements well-known to him before the injury. In which area of the cerebral cortex is the respective center localized? a) Gyrus precentralis b) Gyrus angularis c) Gyrus supramarginalis d) Gyrus lingualis e) Gyrus parahipocampalis 372. A patient has a tumor in the area of the upper nasal meatus. Which cranial nerve's receptors may be affected by the tumor? a)Facial b) Optic c) Adductor d) Trochlear e) Olfactory 373. A patient has impaired vision in the lateral aspects of the visual fields of both eyes (bitemporal hemianopsia). What structure was affected? a) Right optic tract b) Left optic tract c) Optic chiasm d) Retina e) Optic nerves 374. During forensic examination abundant quantity of sweat glands was found in one of the skin shreds. What area did the studied material belong to? a) palm b) abdominal wall c) back d) elbow e) face 375. A patient has neurological symptoms which may be the signs of a stroke in the structure located between the hemispheres. The principal connection between the cerebral hemispheres is: a)pons b)corpus callosum d)vermis d)corpus trapezoideum e) fornix 376. After a trauma to the head, a 49 year-old patient has lost ability to recognize shape and size of objects while touching them (stereognosis). In which areas of the cerebral cortex is the coresponding center normally localized? a) Supramarginal gyrus b) Inferior parietal lobes c) Superior parietal lobes d) parietal gyrus e) Angular gyrus 377. Lack of information from certain parts from the opposite side of the body was noted in the left ventral afferent spinothalamic pathway. Neurons of what nucleus of posterior horn of the spinal cord are affected? a) Substantia spongiosa b) Nucleus proprius c) Substantia gelatinosa d) Thoracic nucleus of Clark e) Ventral nucleus 378. A 45 year-old patient after severe common cold has developed convergent strabismus. Which muscles of the eyeball are damaged? a) Musculus rectus oculi medialis b) Musculus rectus oculi superior c) Musculus rectus oculi lateralis d) Musculus rectus oculi inferior e) Musculus obliqus oculi superior 379. A patient has attacks of chest pain that are accompanied by irradiation of the pain into the left upper limb. What level do somatic pain impulses converge at? a) thalamus b) reticular formation of midbrain c) segments of spinal cord d) sensory zones of hemispheres e) cerebellum 380. A 30 year-old woman has suffered facial nerve neuritis that led to paralysis of the facial muscles and hearing loss. Hearing loss is the result of paralysis of: a) Superior auricle muscle b) Stapedius muscle c) Anterior auricle muscle d) Posterior auriclemuscle e) Nasal muscle 381. In order to differentiate the diagnosis of meningitis, studies of cerebrospinal fluid were conducted. Where in the safe site for a lumbar puncture is? a) L III -L IV b) L II -L III c) L I –L II d) Th XII - L I e) L V - S I 382. A patient has reduced skin sensitivity following a brain trauma. What area of the cerebral cortex may be affected? a) Frontal area of the cortex b) Cingular gyrus c) Postcentral gyrus d) Occipital area e) Precentral gyrus 383. A 58 year-old woman was referred to a doctor with complaints of taste sensitivity impairment. A hemorrhage in the area of medulla oblongata was discovered. Damage of what medulla oblongata nucleus could lead to taste impairment in the patient? a) Nucleus nervi hypoglossi b) Nucleus ambiguus c) Nucleus tractus solitarii d) Nucleus salivatorius inferior e) Nucleus dorsalis nervi vagi 384. The patient has paralysis of the left upper and lower limbs. What center is damaged? a) Motor center of left hemisphere b) Center of simultaneous rotation of the head and eyes to the opposite side of the right hemisphere c) Center of precise movement of the right hemisphere d) Center of stereognosis of the left hemisphere e) Motor center of right hemisphere 385. The surgeon, crossing the pathway of pain sensitivity in the spinal cord (chordotomy), uses dens ligament as a landmark for the procedure. All statements relating to this ligament are true except: a) It is attached to the dura and arachnoid matter b) It extends from pia matter c) It hass about 19-23 fibers d) It separates the right and left parts of subarachnoid space e) It captures the spinal cord, preventing its vertical stretch. 386. A patient was admitted to the hospital with a suspected haematoma of the brain. After a tomographic examination damage of lemniscus lateralis was revealed. What pathway does this structure belong to? a) Tr. Frontopontinus b) Tr. Pyramidalis c) Tr. Acusticus d) Tr. Tectospinalis e) Tr. reticulospinalis 387. After a road accident, a patient has lost the ability to read. In which part of the brain is the lesion located? a) Postcentral gurys b) Angular gyrus c) Precentral gyrus d) Middle frontal gyrus e) Superior temporal gyrus 388. A patient (right-handed) complains of loss of ability to perform fine movements needed to write letters, words, and other signs. Which region of the cerebral cortex is affected? a) Middle part of precentral gyrus from left b) Dorsal part of middle frontal gyrus of left hemisphere c) Middle part of right middle frontal gyrus d) Middle part of right precentral gyrus e) Insula 389. A patient has sustained a trauma of calvaria in a road accident. The patient was admitted to the hospital with severe bleeding and fracture of the skull cap bones. Which of the sinuses of dura mater might be injured? a) sagittalis superior b) petrosus superior c) sigmoideus d) rectus e) transverse 390. A patient complains of vision disturbance. An MRI-image confirmed a tumor of the optic pathway. In case of damage of what of the following parts of conducting pathway of visual analyzer will sensitivity to the light irritants be absent in left-sided parts of retina in both eyes? a) right tractus opticus b) left tractus opticus c) optic chiasma d) right optic nerve e) left optic nerve 391. After a trauma, a 45year-old patient has lost sensitivity in specific areas of the right part of the body. What gyrus of the cerebral hemispheres is affected? a) Inferior temporal b) Middle temporal c) Postcentral d) Precentral e) Superior temporal 392. In a patient with glaucoma increased intraocular pressure is present while secretion of aqueous humor is normal. Damage of what structure of the eyeball is related to disturbance of the outflow of fluid from the anterior chamber? a) Ciliary body b) Vascular layer c) Ciliary muscle d) Venous sinus e) Posterior epithelium of the cornea 393. A 55 year-old unconscious patient was admitted to the emergency department. After MRI examination a hemorrhage in the area next to posterior crus of internal capsule was found. The posterior crus of internal capsule is located between: a)thalamus and globus pallidus b)globus pallidus and nucleus caudatus c)nucleus caudatus and thalamus d)putamen and globus pallidus e) insula and capsula externa 394. A patient was hospitalized with a closed trauma to the head in the area of occipital bone. On examination, gait disorder, balance disorder and hand's tremor were noted. What part of the brain is damaged? a) Cerebellum b) Medulla c) Pons d) Diencephalon e) Spinal cord 395. A 25 year-old man was taken to hospital with skull injuries. An examination of the patient revealed lack of voluntary muscles movements of the head and neck. Damage of which areas of the brain could cause this? a) Lower part of precentral gyrus b) Lower part of postcentral gyrus c) Superior part of precentral gyrus d) Superior part of postcentral gyrus e) Inferior frontal gyrus 396. A patient has convergent strabismus after a severe viral infection. What muscle of the eyeball is shortened? a) rectus medialis b) rectus lateralis c) rectus superior d)rectus inferior e) oblique superior 397. In case of blockage of the median and lateral apertures of IV ventricle occlusive syndrome develops. Where would CSF accumulate in this case? a) III ventricles b) lateral ventricles c) cerebral aqueduct d) Subarachnoid space e) IV ventricles 398. On MRI a tumor at the medial part of crus cerebry base was confirmed. What pathway fibers are normally localized in this area? a) Tr. Occipitotemporoparietopontinus b) Tr. Pyramidalis c) Tr. Frontopontinus d) Tr. Rubrospinalis e) Tr. tectospinalis 399. A patient has aphasia after a haemorrhage (he cannot pronounce words). In what gyrus is the hemorrhage localized? a) Middle temporal b) Superior frontal c) Middle frontal d) Superior temporal e) Inferior frontal 400. A 59 year-old patient has problem with voluntary movement of his right hand because of a tumor of the spinal cord. Nerve impulses that stimulate voluntary muscle contractions are transmitted through: a)funiculus anterior b)funiculus lateralis c)funiculus posterior d)both a) and b) e)both b) and c) 401. In a 12 year-old boy inflammation of the inner ear was complicated by diffuse meningitis. The doctor suggested that the process had spread through communication between the subarachnoid space of the brain and the inner ear perilymphatic space. Through what anatomic formation has the inflammation spreaded? a) Fossa subarcuata b) Hiatus canalis n. petrosi majoris c) Hiatus canalis n. petrosi minoris d) Аqueductus vestibuli e) Fissura petrosquamosa 402. A patient has inflammation of the inner ear. After examination, a lesion of VIII pair of cranial nerves was confirmed by the doctor. Where are the bodies of the first neurons of the auditory analyzer located? a) G. Vestibulare b) G. spirale c) G. Geniculi d) G. Trigeminale e) G. ciliare 403. The patient does not understand the meaning of words, as well as his own speech. What gyrus of the cerebral hemispheres is affected? a) Superior parietal lobe b) Postcentral c) Inferior frontal d) Superior temporal e) Inferior parietal lobe 404. In a 26 year-old patient has lost the ability to read letters, words, and phrases after a surgery. In which area of the cortex the function is not restored? a) Cuneus b) Gyrus anqularis c) Sulcus calcarinus d) Gyrus temporalis superior e) Gyrus supramarginalis 405. A 22 year-old patient was admitted to the neurological department. An increased pupil diameter and distuirbance of pupillary reflex was noted by the doctor during examination. Activity of what muscle is blocked? a) Musculus dilatator pupillae b) Musculus sphincter pupillae c) Musculus ciliaris d) Musculus rectus superior e) Musculus rectus inferior 406. A patient has developed autonomic disorders, such as sleep disturbance, thermoregulation and metabolic disfunction, as well as diabetes insipidus. Irritating of nuclei of what areas ofs brain caused these symptoms? a) Hypothalamus b) Tegmentum of the midbrain c) Pons d) Peduncles of the brain e) Medulla 407. A 49 year-old patient complains of dark spots appearance before his eyes. Upon ophthalmoscopy swelling of retina in the most sensitive to light area was found. What area of retina was affected? a) Fovea centralis b) Discus nervi optici c) Ora serata d) Macula lutea e) Pars caeca retinae 408. In a year-old patient 49 on MRI a tumor of the pons, particularly in tegmentum involving fibrae pontis transversae, was found. What pathway do the fibers belong to? a) Tr. Tectospinalis b) Tr. Rubrospinalis c) Tr. Pyramidalis d) Tr. Corticopontocerebellaris e) Tr. reticulospinalis 409. Examination of the brain using MRI has revealed a hematoma in the genu of internal capsule. What pathway is damaged in this case? a) Tr.cortico-spinalis b) Tr.cortico-nuclearis c) Tr.cortico-fronto-pontinus d) Тr.cortico-thalamicus e) Tr.thalamo-corticalis 410. A patient has a tumor of brainstem. Because of this, he has strabismus, paresis of facial muscles, lower jaw dropping, hearing impairment. In what part of the brainstem is the tumor located? a) Spinal cord b) Medulla c) Midbrain d) Cerebellum e) Pons 411. In a 45 year-old patient voice changes and difficulty swallowing are observed due to tumor in medulla, his food gets to his nasal cavity while eating. What nucleus of medulla oblongata is damaged by the tumor? a)solitarius b)spinalis n.trigemini c)ambiguus d)dorsalis n.vagi e) hypoglossy 412. Upon examination of a patient, an injury to lemniscus lateralis was confirmed. What pathway does this structure belong to? a) Tr. acusticus b) Tr. pyramidalis c) Tr. frontopontinus d) Tr. tectospinalis e) Tr. reticulospinalis 413. On MRI a tumor in the posterior horn of spinal cord was found. Bodies of 2nd neurons of tractus spinocerebellaris posterior are situated in: a) nucleus thoracicus b) nucleus intermediomedialis c) nucleus intermediolateralis d) nucleus proprius of posterior horn e) nucleus marginalis 414. In a 65 year-old patient a hemorrhage in the anterior horn of the spinal cord was diagnosed. What is the function of the aterior horns? a) Motor b) Sensory c) Sympathetic d) Parasympathetic e) Mixed 415. After an inflammatory disease in a 22 year-old patient incomplete sideways movement of the eyeball was noted. What nerve was damaged? a) Trochlear b) Abducens c) Optic d) Oculomotor e) Facial 416. A patient with a stroke has damage to the posterior crus of the internal capsule. What pathway is located there? a) Radiatio acustica et radiatio optica b) Tr. Corticonuclearis c) Tr. Corticospinalis d) Tr. Parietopontinae e) Tr. occipitopontinae 417. During surgery to remove a tumor a doctor manipulates on the medial wall of the tympanic cavity. Destruction of what canal of temporal bone is possible? a) Tympanic b) Facial c) Carotid d) Musculotubular e) Chorda tympani 418. A patient's pupil becomes narrowed after intoxication; when the lighting is reduces, the pupil does not expand. What part of the central nervous system was injured by the toxin? a) Base of the midbrain's cerebral peduncle near the inferior colliculi of the tegmentum b) Lateral horn of the spinal gray matter on the level of C VIII — ThI c) Lateral geniculate body of the diencephalon d) Tectum of midbrain on the level of superior colliculi of tegmentum e) Superior colliculi of tegmentum 419. A 45 year-old patient was admitted to the emergency department due to cerebral haemorrhage. After initial stabilization of the patient, damage to subcortical centers of equilibrium was confirmed. Where is this center located? a) Hypothalamus b) Pons c) Thalamus d) Basal nuclei e) Nuclei of cerebellum 420. A 47 year-old patient has a brain tumor. On X-ray expansion of I-st, II-nd and III- d ventricles has been revealed. Indicate the most probable location of the tumor. a) Telencephalon b) Medulla c) Pons d) Midbrain e) Cerebellum 421. A patient has cerebral hemorrhage that nextended into the third ventricle of the brain. What structures take part in the formation of 3rd ventricle posterior wall? a) septum pellucidum b) lamina terminalis c) corpus fornicis d) stria medullaris e) trigonum colaterale 422. A 62 year-old patient after a trauma to the head has a hematoma in the area of gyrus angularis. The patient can not read or understand written text, but his visual function is not impaired. The nucleus of which analyzer was affected? a) Nucleus of visual analyzer of writing speech b) Nucleus of sound analyzer of oral speech c) Nucleus of motor analyzer of writing speech d) Nucleus of visual analyzer e) Nucleus of motor analyzer of oral speech 423. In a patient after a stroke, a hemorrhage in the area of the medial surface of the occipital lobe of the brain was detected. The function of which analyzer is likely to be impaired? a) Gustatory b) Auditory c) Visual d) Olfactory e) Somatosensory 424. As a result of neurosurgical intervention in an animal, there is narrowing of pupils, paralysis of accommodation, falling back of the eyeball. What neural centers were injured? a) parasympathetic centers of midbrain b)superior colliculi of tectum c) lateral geniculate bodies d) C8-Th1 segments of spinal cord e) red nucleus 425. A patient complains of wobbly gait and trouble walking. During examination of the patient a disturbance of proprioceptive sensation was found. Bodies of 2nd neurons of tractus spinocerebellaris anterior are situated in: a)nucleus thoracicus b)nucleus intermediomedialis c)nucleus intermediolateralis d)nucleus proprius of posterior horn e) nucleus marginalis 426. A year-old patient 45 suffered severe cerebrovascular stroke. After initial stabilization of the patient, there is loss of ability to clearly pronounce words. Damage of what areas of the cerebral cortex has caused the impairment of motor speech center? a) Supramarginalis. b) Gyrus frontalis inferior. c) Precentralis. d) Angularis. e) Temporalis superior. 427. In a 10 year-old child, nasopharyngitis was complicated by acute otitis. The middle ear is separated from the inner ear by: a)round window b)tympanic membrane c)oval window d)both a and c e)both a and b 428. A 38 year-old patient was admitted to the neurological department with a hemorrhage in the area near the red nucleus, substantia nigra and corpora quadrigemina. In which region of the brain are corpora quadrigemina, nucleus ruber and substantia nigra located? a) diencephalon b) metencephalon c) mesencephalon d) myelencephalon e) cerebellum 429. A 50 year-old patient after a stroke (hemorrhage in the brainstem) is not responding to unexpected or excessive sound stimuli. The doctor suspected that subcortical sound centers were damaged; the centers represent the beginning of a protective pathway involved in deflection movements of the body in case of danger signals. Where are these subcortical centers of hearing located? a) Colliculus superior b) Corpus geniculatum mediale c) Brachium colliculi superiorіs d) Brachium colliculi inferioris e) Colliculus inferior 430. A two year-old child’s viral infecton was complicated by constant tearing. Upon examination, there is blockage of ducts, conducting tear from the orbit to the nasal cavity. What is this channel called? a) superior lacrimal canal b) nasolacrimal duct c) inferior lacrimal canal d) rivus lacrimalis e) lacus lacrimalis 431. A 60 year-old woman presented to a clinc with complains of difficulty moving the tongue that prevents her from eating and talking normally. Examination of the brain using MRI showed that the patient has haemorrhage in the lower part of medulla oblongata. Damage of what medulla oblongata nucleus might be suspected? a) Nucleus of b) Inferior salivatory nucleus c) Nucleus of accessory nerve d) Nucleus ambiguus e) Nucleus of solitary tract 432. Vasospasm of the inner ear's vessels leads to stimulation of vestibular part of VIII pair of cranial nerves, and such symptoms as dizziness, nausea, impaired balance, and nystagmus appear. Where are receptors of pars vestibularis n. vestibulocochlearis located? a) Ganglion oticum b) Organum spirale c) Ganglion spirale cochleae d) Cristae ampullaris ductus semicircularis, macula sacculi, macula utriculi e) Ganglion vestibulare 433. A patient complains of the headache, dizziness, and balance disturbance. Obstruction of connection between third and fourth ventricles was found. Fourth ventricle is located within: a) cerebrum b) mesencephalon c) rhombencephalon d) diencephalons e) spinal cord 434. In a patient a cerebral hemorrhage has expanded into the third ventricle of the brain. What structures take part in formation of 3rd ventricle anterior wall? a) septum pellucidum b) lamina terminalis c) corpus fornicis d) stria medullaris e) thalamus 435. A 35 year-old man after meningoencephalitis has sharp decline in hearing. Examination has not revealed any pathology of sound conduction or sound perception parts of the organ of hearing. In what cortical gyrus are pathological changes taking place? a) supramarginalis b) middle temporal c) superior frontal d) superior temporal e) insula 436. A patient complaints of failure to understand words, but he hears the sounds. What cortical analyzer is most likely damaged? a) Nucleus of sound analyzer b) Motor analyzer of writing speech c) Optical analyzer of writing speech d) Sound analyzer of oral speech e) Motor speech center 437. A patient was admitted to the hospital after an appointment with endocrinologist. A deformity of the Turkish saddle’s dorsum was revealed on his skull X-ray. A growth of what structure within the brain could have caused these findings? a) Hypothalamus b) Epiphysis c) Hypophysis d) Optic chiasm e) Thalamus 438. A 45 year-old patient has impaired feeling of taste after a severe common cold. What nucleus of medulla oblongata is damaged by intoxication? a) solitarius b) spinalis n.trigemini c) ambiguus d) dorsalis n.vagi e) facialis 439. A 40 year-old man was admitted to an ophtalmology department; he suffered a burn to his eye two weeks ago. What structure was affected? a) Iris b) Ciliary body c) Cornea d) Lens e) Corpus vitreum 440. A 60 year-old woman was admitted to the hospital with a tumor in the tegmentum of midbrain within the area of lemniscus medialis. What of the pathways is this structure a part of? a) Tr. Pyramidalis b) Tr. Bulbothalamicus c) Tr. Frontopontinus d) Tr. Rubrospinalis e) Tr. reticulospinalis 441. A forensic pathologist examined a section of the head of a murder victim. Which of the following is not a lobe of the brain? a) parietal b) temporal c) sphenoid d) occipital e) frontal 442. A brain tumor near the red nucleus was revealed on CT. What part of the brain is damaged by the tumor? a) Midbrain b) Cerebellum c) Diencephalon d) Medulla e) Pons 443. In a patient, inflammation of a part of the eyeball, where blood vessels are absent, was diagnosed. What structure is characterized by this particular feature? a) cornea b) vascular coat c) iris d) ciliary body e) retina 444. A 10 year-old child with nasal inflammation complains of a pain in his left ear. The examination has found inflammation of the middle ear. Through which anatomical formation did the infection spread into the middle ear? a) Semicanal of auditory tube b) Tympanic canal c) Musculotubal canal d) Carotic canal e) Semicanal of tensor tympany muscle 445. A patient was admitted to hospital with complaints of balance disturbance. In the erect position with his eyes closed the patient loses balance and almost falls. What part of brain is most likely damaged? a) basal nuclei b) cerebellum c) thalamus d) gyrus precentralis e) fornix 446. A 69 year-old patient complaints of sensory loss in the lower left part of his body and in the lower part of his leg. Bodies of 2nd neurons of tractus spinothalamicus are situated in: a)nucleus thoracicus b)nucleus intermediomedialis c)nucleus intermediolateralis d)nucleus proprius of posterior horn e) substantia gelatinosa 447. A 52 year-old man has limbic form of Alzheimer's disease, the main manifestation of which is progressive memory loss, that is related to the limbic system lesions. What structures are affected? a) Fornix, corpora mamillaria, substantia innominata b) Lobus limbicus c) Bulbus olfactorius, tractus olfactorius d) Gyrus fornicatus e) Cornu Ammonis 448. In Reiter's disease there is a simultaneous damage to eyes, urinary tract and joints. The most often loccaliizaton of eye damage is the vascular layer. What structures belong to the vascular layer? a) Iris, lens, corpus ciliare b) Cornea, iris, corpus ciliare c) Iris, corpus ciliare, sclera d) Corpus vitreum, iris, corpus ciliare e) Iris, corpus ciliare, choroidea 449. A 34 year-old patient was admitted to the hospital with compression fracture at the level of the fourth thoracic vertebra. What spinal cord segment will be damageds? a) ThVII b) ThII c) ThV d) LI e) CVIII 450. In a 67 year-old patient, as a result of a sudden circulatory disorder in the region of a. cerebri posterior, lesions of the posterior hypothalamic area, Lewis' body, dentato- rubro-thalamic pathway were diagnosed. Through which structures does the cerebello- rubro tract pass? a) Inferior cerebellar peduncles b) Middle cerebellar peduncles c) Superior cerebelar peduncles d) Pedunculi cerebri e) pedunculi flocculi 451. A patient has lost the ability to recognize the objects by their characteristic sounds (clock ringing, music playing) after a head trauma. What part of the brain is damaged? a) Insula b) Lobus frontalis c) Lobus parietalis d) Lobus occipitalis e) Lobus temporalis 452. A 70 year-old woman was diagnosed with a tumor on the border of tectum and the basis of crus of the midbrain. Which of the internal structures of midbrain is located in this area? a) Substantia nigra b) Nucleus ruber c) Tr. Frontopontinus d) Lemniscus medialis e) Tr. reticulospinalis 453. In an animal (a cat) an experimental neurosurgical interference was performed, as a result of which the cat has lost ability to accommodate and converge her eyes. What nervous structures were damaged? a) ganglion superius trunci sympathetici b) parasympathetic centers of medulla oblongata c) tectum mesencephali d) thalamus e) pons 454. A woman presented to a clinc with complaints of taste sensitivity disturbance in her tongue. Damage of which nucleus of the medulla oblongata could cause this problem? a) Solitarius b) Ambiguus c) Hypoglossal nerve nucleus d) Inferior salivatory e) Dorsal nucleus of vagus 455. A69 year-old woman was admitted to the hospital complaining of chronic headache. In the study using magnetic resonance imaging, a neurologist has diagnosed tumors in the area of interpeduncular fossa. What coveres this formation? a) tectum mesencephali b) substantia grisea centralis c) corpus geniculatum mediale d) corpus pineale e) substantia perforata posterior 456. An examination of the patient revealed lack of vision in medial halves of the visual fields of both eyes. What part of the optic pathway is most likely damaged? a) Chiasma opticus b) N. Opticus. c) Tractus opticus. d) Sulcus calcarinus. e) Corpus geniculatum laterale 457. After a severe trauma of the back sustained in a car accident, a patient was reffered to an MRI examination. Damage of anterior funiculus of spinal cord was found. Anterior funiculus of spinal cord contains the following motor tracts, except: a)tr.vestibulospinalis b)tr.reticulospinalis c)tr.rubrospinalis d)tr.tectospinalis e) tr. Coricospinalis anterior 458. A 22 year-old patient was admitted to the neurosurgical department after a car accident with a brain injury. A lesion of the posterior part of left inferior frontal gyrus was diagnosed. Thus, the function of which analyzer center is disturbed? a) Motor center of oral speech b) Somatosensory center c) Motor center d) Motor center of writing speech e) Sound analyzer 459. In a 15 year-old patient nasopharyngitis was complicated by purulent otitis and the pus penetrated upper wall of tympanic cavity. What cranal fossa did the pus spread to from tympanic cavity? a) Orbit b) Anterior cranial fossa c) Middle cranial fossa d) Posterior cranial fossa e) Pterigopalate fossa 460. An 8 year-old child with suppurative middle ear otitis had the infection spread to the jugular bulb. This complication developed because of a thinning of the walls of tympanic cavity. Anomaly of which wall was present? a) Anterior b) Medial c) Superior d) Lateral e) Inferior 461. A 49 year-old patient has damage of white matter of the spinal cord within the posterior cord (funiculus) and clinic disfunction of one of the pathways of the spinal cord. What comprises the posterior cord (funiculus) of the spinal cord? a) fasc.spinocerebellaris anteior b) fasc. Gracilis c) fasc. Cuneatus d) fasc. Spinocerebellaris posterior e) fasc. gracilis et fasc. Cuneatus 462. Patient M., who is 41 years old, was admitted into infectious disease department of a hospital with high fever. Meningeal symptoms are noted upon eamination. A spinal puncture was performed. Which anatomical formation has been punctured? a) spatium subarachnoideum b) spatium subdurale c) spatium epidurale d) cavum trigeminale e) cisterna cerebellomedullaris posterior 463. A 12 year-old patient with mumps suddenly develops unilateral complete deafness associated with acute suppurative labyrinthitis. Where are receptors of pars cochlearis of n. vestibulocochlearis located? a) Macula utriculi b) Ganglion spirale cochleae c) Organum spirale d) Macula sacculi e) Cristae ampullaris ductus semicircularis 464. A 54 year-old c patient complains of vision problem. Upon examination disturbance of the process of accommodation of the eye was revealed. What muscle is damaged? a) Musculus ciliaris b) Musculus dilatator pupillae c) Musculus rectus superior d) Musculus sphincter pupillae e) Musculus rectus inferior 465. A patient after a stroke can't write, but is able to understand speech and to read. Where is the lesion located? a) Gyrus precentralis b) Gyrus frontalis superior c) Gyrus frontalis medius d) Gyrus postcentralis e) Gyrus temporalis superior 466. A 65 year-old man was admitted to the neurological department with the diagnosis of stroke. Upon examination, the patient's facial and masticatory muscles are paralysed on the left. An MRI showed a haematoma in the genu of internal capsule on the right side. Which pathway was damaged? a) Tr. cortico-temporo-parieto-occipito-pontinus b) Tr. Cortico-spinalis. c) Tr. Cortico-thalamicus. d) Tr. Cortico-ponto-pontinus. e) Tr. Cortico-nuclearis. 467. A patient has autonomous nervous system disfunction due to mushrooms poisoning. Sympathetic centers form a continuous cellular column, that extends through the following segments of the spinal cord: a)C6 – Th9 b)C8 – L2 c)C3 – S3 d)C7 – L5 e) S2-S4 468. In a 75 year-old t patient a stroke in the area near the red nucleus was diagnosed. Red nucleus is an important part of: a) pyramidal system b) extrapyramidal system c) limbic system d) rhinencephalon e) medulla 469. A patient complaints of taste disturbance. However, touch, pain and temperature sensitivities are intact. What papillae of the tongue have no taste receptors? a) filiform b) vallate c) fungiform d) foliate e) both a and b 470. A laboratory experiment on a dog was conducted to study central parts of auditory system. One of the mesencephalon structures was destroyed. The dog has lost the orienting response to auditory signals. What structure was destroyed? A. Superior colliculi of corpora quadrigemina. B. Inferior colliculi of corpora quadrigemina. C. Substantia nigra. D. Reticular formation nuclei. E. Red nucleus. 471. During experiment a dog has developed conditional digestive reflex in response to a sound stimulus. This conditional reflex will not be exhibited anymore after extirpation of the following areas of the cerebral hemispheres: A) Occipital lobe on one side. B) Parietal lobe on both side. C) Temporal lobe on one side. D) Temporal lobe on both side. E) Occipital lobe on both side. 472. After a craniocerebral injury a patient has lost the ability to recognize shapes of object by touch (stereognosis). What area of the cerebral cortex normally contains relevant centers? A) Inferior parietal lobule. B) Superior parietal lobule. C) Supramarginal gyrus. D) Angular gyrus. E) Postcentral gyrus. 473. After a traffic accident a 36 year-old patient has developed muscle paralysis of the extremities on the right, lost pain and temperature sensitivity on the left, and partially lost tactile sensitivity on both sides. What part of the brain is most likely damaged? A) Motor cortex on the left. B) Rightside of the spinal cord. C) Left side of the spinal cord. D) Posterior horn of the spinal cord. E) Anterior side of the spinal cord.

331. b 332. a 333. e 334. c 335. d 336. a 337. a 338. a 339. a 340. a 341. e 342. a 343. d 344. c 345. b 346. c 347. a 348. d 349. e 350. a 351. d 352. a 353. e 354. d 355. b 356. a 357. b 358. a 359. a 360. c 361. b 362. b 363. b 364. b 365. e 366. e 367. d 368. a 369. d 370. a 371. c 372. e 373. c 374. a 375. b 376. c 377. b 378. c 379. c 380. b 381. a 382. c 383. c 384. e 385. d 386. c 387. b 388. b 389. a 390. b 391. c 392. d 393. a 394. a 395. a 396. a 397. d 398. c 399. e 400. d 401. d 402. b 403. d 404. b 405. b 406. a 407. d 408. d 409. b 410. e 411. c 412. a 413. a 414. a 415. b 416. a 417. b 418. a 419. e 420. d 421. e 422. a 423. c 424. a 425. b 426. b 427. d 428. c 429. e 430. b 431. a 432. d 433. c 434. b 435. d 436. d 437. c 438. a 439. c 440. b 441. c 442. a 443. a 444. a 445. b 446. d 447. e 448. e 449. c 450. c 451. e 452. a 453. c 454. a 455. a 456. a 457. c 458. a 459. c 460. e 461. e 462. a 463. c 464. a 465. c 466. e 467. b 468. b 469. a 470. b 471. d 472. b 473. b

PNS and PVS

474. A patient, following an injury, complained of decreased pain and temperature sensitivity in some of his fingers, particularly 1 1/2 on the palmar surface and of 2 1/2 on the dorsal one, including the pinky finger. What nerve is damaged? A. N. medianus B. N. radialis C. N. ulnaris D N. musculocutaneus E. N. cutaneus antebrachii medialis 475. A patient complains of neurologic symptoms: he neither can lift his eyebrow on the left, nor close his eyes completely, nor bare his teeth. What nerve is affected? A. maxillary B. optic C. facial nerve D. mandibular E. oculomotor 476. Upon examination, a patient was diagnosed with a lesion in the dorsal part of pons, his chewing function is impaired. Nucleus of what nerve is affected? A. The nucleus of hypoglossal nerve B. Motor nucleus of the facial nerve C. Pontine nucleus of trigeminal nerve D. Motor nucleus of trigeminal nerve E. Ambiguus nucleus of the vagus nerve 477. A patient after a fracture of the upper third of humerus developed paralysis of posterior muscles of shoulder and forearm. What nerve is damaged? A. N. ulnaris B. N. radialis C. N. medianus D N. musculocutaneus E. N. cutaneus antebrachii medialis 478. A physician carries out auscultation of one of the valves of heart, placing the membrane of stethoscope in the area of the second intercostal space on the right. Which valve is being auscultated? A. right atrioventricular B. aortic C. pulmonary D. left atrioventricular E. tricuspid 479. Following a colds, a patient has numbness in the right side of his face. Examination revealed decreased pain and temperature sensitivity in the right side of the face. What nerve is damaged? A. Hypoglossal B. Facial C. Glossopharyngeal D. Vagus E. Trigeminal 480. A patient has lost skin sensation in the anterior region of his thigh. Branches of what nerve were injured? A. Genitofemoral nerve B. Obturator nerve C. Sciatic nerve D. Ilioinguinal nerve E. Femoral nerve 481. A patient has swelling, enlargement of veins and formation of venous nodes on the medial surface of the thigh. What veins are affected? A. V. iliaca externa B. V. saphena parva C. V. femoralis D. V. poplitea E. V. saphena magna 482. A patient complains of pain in the area of diaphragm during breathing; the pain occurred after a chest injury. What nerve is affected? A. Supraclavicular nerve B. intercostal nerves C. phrenic nerve D. suprascapular nerve E. transverse nerve of neck 483. A girl complains of difficulty extending her fingers and hands, loss of skin sensitivity on posterior surface of shoulder, forearm and I-III fingers. What nerve is affected? A. N. musculocutaneus B. N. ulnaris C. N. medianus D N. radialis E. N. cutaneus antebrachii medialis 484. A 63 year-old patient was admitted to hospital with esophageal bleeding. Liver cirrhosis and worsening of blood flow in the system of vena porta is confirmed. What vein carries the blood from the system of vena porta to the system of superior vena cava through esofageal veins? A. v.paraumbilicalis B. v.gastrica dextra C. v.lienalis D. v.gastrica sinistra E. v.epigastrica inferior 485. Patient M., aged 43, is seen by neurologist due to complaints of shortness of breath and chest pain, difficulty while coughing, and hiccups. What nerves are affected? A. intercostal B. internal n.vagus D. sympathetic trunk E. diaphragmatic 486. When swallowing food, a patient noted difficulty that is associated with paralysis of the soft palate. What nerve is damaged? A. II branch of trigeminal nerve B. facial nerve C. I branch of trigeminal nerve D. III branch of trigeminal nerve E. hypoglossal nerve 487. A 45 year-old patient complains of loss of sensation in the area of posterior 1/3 of the tongue. Function what pair of cranial nerves is impaired? A. Х B. ІХ C. VIII D. V E. XII 488. A patient has atrophy of posterior group of shin muscles. What nerve is affected? A. Tibial nerve B. Superficial peroneal nerve C. Deep peroneal nerve D. Femoral nerve E. obturator nerve 489. A physician carries out auscultation of one of the heart valves in the area of the heart apex. What valve is being auscultated? A. bicuspid B. right atrioventricular C. aortic D. pulmonary E. tricuspid 490. After the injured soft tissues of the oral cavity, the patient lost taste sensitivity of the posterior third of the tongue. What nerve is affected? A. N. facialis B. N. glossopharyngeus C. N. hypoglossus D. N. lingualis E. Сhorda tympani 491. A patient has difficulty chewing. Partial atrophy of masticatory muscles located below zygomatic arc is present. Branches of what nerves innervate aforementioned muscles? A. N. infraorbitalis B. N. maxillaries C. N. alveolaris inferior D. Nn. alveolares superiors E. N. mandibularis 492. A 50 year-old patient with pain, weakness and cyanosis of the upper extremity is seen in a clinic. Swelling in the is noted upon examination, as well as hypertrophy of anterior scalenus muscle that compresses neurovascular bundle. In what topographic space is the artery compressed? A. previsceral B. suprasternal C. antescalenum D. retrovisceral E. interscalenum 493. The patient complains of a dry area under the tongue on the right side. The doctor found that chorda tympani was compressed at its entry to the right infratemporal fossa through fissura: A. Petroocipitalis B. Tympanomastoidea C. Petrosguamosa D. Sphenopetrosa E. Petrotympanica 494. An 18 year-old patient has an appointment with neurologist due to complaints of inability to stand on his toes. Atrophy of gastrocnemius muscle and impaired skin sensitivity in the soles were noted upon examination. Function of what nerve is impaired? A. N. cutaneus femoris lateralis B. N. fibularis C. N. femoralis D. N. tibialis E. N. Saphenus 495. A patient, following fracture of radiu,s developed traumatic neuritis, increasing swelling of hand, limitation of fingers flexion, paresthesia, increased sweating. What nerve is damaged? A. median nerve B. radial nerve C. ulnar nerve D. axillary nerve E. interosseus nerve 496. A 30 year-old male is seen by neurologist due to impaired skin sensitivity on the posterior surface of the right leg in its middle and lower thirds. What nerve is damaged? A. suralis B. tibialis C. saphenus D. cutaneus femoris posterior E. femoralis 497. An examination of a patient revealed ptosis, divergent strabismus, mydriasis, and limited mobility of the eyeball. What nerve is damaged? A. N.trochlearis B. N.abduceus C. N.opticus D. N.oculomotorius E. N.ophthalmicus 498. A patient has difficulty chewing. Atrophy of right temporalis muscle is noted upon examination. While opening the mouth, the jaw deviates to the left. What a nerve is affected? A. Motor portion of the mandibular B. Facial C. inferior alveolar D. maxillary E. Mylohyoid 499. A patient with hemorrhagic stroke has slurred speech. His larynx and movements of the mandible are preserved. What nerve nuclei are affected by the hemorrhage? A. Nuclei n. glossopharyngeus B. Nuclei n. vagi C. Nuclei n. accessorii D. Nuclei n. facialis E. Nuclei n. hypoglossi 500. A patient suffered myocardial infarction of the posterior wall of the right ventricle. What artery branches are affecteded? A. left subclavian B. right coronary C. left coronary D. right subclavian E. common carotid 501. A patient has impaired vision in the lateral halves of the visual fields of both eyes (bitemporal hemianopsia). What nerve structure is damaged? A. left optic tract B. optic chiasm C. right optic tract D. retina E. optic nerve 502. A patient has tearing and increased salivation. This condition, combined with other symptoms of irritation is regarded as one associated with some fibers of cranial nerves. What are these fibers? A. parasympathetic fibers of the facial nerve B. parasympathetic fibers of the oculomotor nerve C. somatic motor fibers of the oculomotor nerve D. parasympathetic fibers of the vagus nerve E. somatic motor fibers of the facial nerve 503. Developmentally aortic arch is homologous to: A. left carotid artery B. left subclavian artery C. truncus brachiocephalicus D. right carotid artery E. right subclavian artery 504. A patient after a surgery noticed reduced sensitivity on the anterior and lateral surface of his neck. What nerve provides sensory innervation for this area of the neck? A. Nn. Supraclaviculares B. N. Auricularis magnus C. N. transversus colli D. N. occipitalis minos E. N. phrenicus 505. A physician carries out auscultation of one of the heart valves in the second intercostal space on the left. What valve is being auscultated? A. pulmonary B. right atrioventricular C. aortic D. left atrioventricular E. tricuspid 506. A 36 year-old patient examined by a neurologist was diagnosed with loss of skin and pain sensitivity in the superciliary region of the forehead, cheeks and chin on the right. Function of what nerve is impaired? A. N. abducens B. N. facialis C. N. oculomotorius D. N. trochlearis E. N. trigeminus 507. A patient with stabbed wounds of the right hand has loss of sensitivity on the lateral dorsal side of the hand and proximal phalanges of the 1st, 2nd and 3rd fingers. What nerve is damaged? A. N. radialis B. N. ulnaris C. N. medianus D. N. musculocutaneus E. N. cutaneus antebrachii medialis 508. A patient has no sensation in the front triangle of the neck. What nerve of the cervical plexus is damaged? A. minor occipital B. major auricular C. transverse cervical D. supraclavicular E. cervical loop 509. A patient complains of decreased sensitivity of the skin in the medial dorsal and palmar surfaces of the hand. What nerve is damaged? A. N. cutaneus antebrachii medialis B. N. radialis C. N. medianus D N. musculocutaneus E. N. ulnaris 510. A patient is seen in an emergency room with fractures of hand bones accompanied by heavy bleeding of the . What vessel is damaged? A. a.brachialis B. a.profunda brachii C. a. ulnaris D. a.radialis E. a. axillaries 511. A 40 year-old man is undergoing lymphography of organs of the thoracic cavity; a tumor of an organ is confirmed. Lymphatic vessels of the organ drain directly into ductus thoracicus. What organ is this? A. liver B. trachea C. heart D. pericardium E. esophagus 512. A patient sustained a trauma of the elbow with damage to the medial epicondyle of humerus. Define what nerve might be damaged. A. radialis B. axilaris C. ulnaris D. medianus E. musculocutaneus 513. A 40 year-old patient has disfunction of the sublingual and submandibular salivary glands on the left following a maxillofacial trauma. Saliva from these glands is hardly excreted. Function of what nerve is impaired? A. XI B. VI C. Х D. XIІ E. VII 514. A 54 year-old man complains of lack of sensitivity of the lower eyelid, lateral surface of the nose and upper lip. Inflammation of the second branch of trigeminal nerve is confirmed. Through what foramen does the branch exit the skull? A. foramen rotundum B. foramen lacerum C. foramen ovale D. foramen spinosum E. superior orbital fissure 515. In a patient with aneurysm of the right subclavian artery has hoarseness of voice. What nerve might be involved? A. N. laringeus reccurens dexter B. N.laringeus superior dexter C. N.laringeus reccurens sinister D. N.laringeus superior sinister E. N. laringeus inferior sinister 516. A 35 yea-old female complains of slurring of speech and hoarseness that occurred after surgical removal of the thyroid gland. Damage of what nerve during the surgery could cause these symptoms? A. hypoglossal nerve B. superior laryngeal nerve C. recurrent laryngeal nerve D. lingual nerve E. mandibular nerve 517. A patient complains of increased pain sensitivity of auricle and ear canal. Palpation behind the sternocleidomastoid muscle causes considerable pain. Irritation of the what nerves can cause these signs? A. Nn. Supraclaviculares B. N. transversus colli C. N. occipitalis minor D. N. auricularis magnus E. N. vagus 518. A patient has an inflammatory purulent process of skin of the first interdigital space of his foot. What lymph nodes are regional for indicated area and react with pain and swelling? A. Superficial inguinal B. Deep inguinal C. Internal iliac D. Superficial iliac E. Common iliac 519. A patient is diagnosed with a fracture in the middle third of humerus with damage of radial nerve. What artery is most likely to be damaged? A. Anterior humeral circumflex artery B. Deep artery of the arm C. Posterior humeral circumflex artery D. Superior lateral ulnar artery E. Inferior ulnar collateral artery 520. A patient with infectious parotitis developed paralysis of facial muscles and dropping of the right corner of his mouth. What could have cause these symptoms? A. Right-side neuritis of the oculomotor nerve B. Left-sided neuritis of facial nerve C. Right-sided neuritis of the trigeminal nerve D. Left-sided neuritis of the trigeminal nerve E. Right-sided neuritis of facial nerve 521. A patient has dropping of the right shoulder and right scapula. What nerve is damaged? A. Radial right B. Subscapular right C. Axillary right D. Accesorius right E. Suprascapular right 522. A 62 year-old female complains of pain in the hip joint during movement, and pain in the medial thigh muscles. Damage of what nerve could have caused this? A. obturator nerve B. femoral nerve C. Sciatic nerve D. Ilioinguinal nerve E. Genitofemoral nerve 523. In a patient, extension of hip joint is impaired, as is flexion of knee joint, and all foot and ankle movements are lost. What of the following nerves is damaged? A. femoral B. tibial C. obturatorius D. sciatic E. common fibular 524. A neurologist noted that a patient lost sensitivity in his foot and lateral surface of the V toe. What nerve innervates this area of the foot? A. N. cutaneus surae lateralis B. N. Saphenus C. N. fibularis (peroneus) superficialis D. N. fibularis (peroneus) profundus E. N. suralis 525. A patient came to a doctor with a complaint of difficulty chewing. Examination revealed atrophy of temporal and masseter muscles on the right. When the patient opens his mouth, the jaw deviates to the left. What nerve is affected? A. The motor part of mandibular B. Facial C. Inferior alveolar D. Maxillary E. Mylohyoid 526. A patient has tissue ischemia below knee joint, accompanied by “intermittent claudication” (pain during walking that is relieved by rest). What artery is occluded in this case? A. popliteal. B. External iliac. C.Fibular. D. Deep artery of the thigh. E. Descending genu 527. A 45 year-old patient has pale lower leg skin, pulsation of dorsal foot and posterior tibial arteries is absent. Femoral artery pulsaton is preserved. What artery is affected? A. deep artery of the thigh B. external iliac C. fibular D. popliteal E. A. descending genu 528. After a surgery, a patient has reduced sensitivity on the anterior and lateral surface of the neck. What nerve ensures sensitivity of this area? A. N. occipitalis minos B. N. auricularis magnus C. Nn. Supraclaviculares D. N. transversus colli E. N. Phrenicus 529. A patient was diagnosed with a lesion of the third branch of trigeminal nerve. Through what opening of the skull does this branch come out? A. superior orbital fissure B. foramen lacerum C. foramen rotundum D. foramen spinosum E. foramen ovale 530. A patient complains of face asymmetry. The doctor noted that on the left side the eyebrow is lower than on the right, there are no forehead creases, the left eye is semi- closed, the eyeball protrudes forward. What nerve is affected? A. I pair B. VII pair C. V pair D. VI pair E. IV pair 531. A nurse injected a medication into the muscles on the back surface of the shoulder. Suddenly the patient felt pain in the muscles of the shoulder, that irradiated into the back surface of the forearm. What nerve was damaged? A. axillary nerve B. ulnar nerve C. median nerve D. radial nerve E. musculocutaneous nerve 532. While examining a 70 year-old patient, the doctor decided to check pulse on his foot. What artery is most suitable for this purpose? A. A . tibialis posterior B. A. dorsalis pedis C. A. tibialis anterior D. A. plantaris medialis E. A. plantaris lateralis 533. A patient has diminished skin sensitivity on the medial surface of the forearm. What nerve is damaged? A. medial cutaneous nerve of forearm B. medial cutaneous nerve of arm C. radial nerve D. ulnar nerve E. axillary nerve 534. A 44 year-old patient sustained traumatic rupture of tendons and superficial blood vessels of his left palm. Following surgery and removal of necrotic muscle tissue, blood flow was restored. What vessels were the sourse of blood supply? A. Aa. perforantes B. Arcus palmaris superficialis C. Aa. digitales palmares communes D. Aa. metacarpeae palmares E. Arcus palmaris profundus 535. A patient was diagnosed with the ischemic osteochondropathy of the head of femur. What artery is damaged? A. a.femoralis B. a. profunda femoris C. a.obturatoria D. a.iliaca externa E. a.fibularis 536. Upon examination of a young man with a stabbed wound of external surface of knee below the head of fibula it was noted that his foot hangs down, toes are flexed, dorsal flexion of the affected foot is impossible; while walking, the patient steps first on his toes, then on the lateral side of his foot, and then on his sole; sensitivity on the lateral side of leg and foot is diminished. What nerve is damaged? A. tibialis B. peroneus communis C.femoralis D. peroneus superficialis E. peroneus profundus 537. A patient complains of sharp pain in the face. What nerve is affected? A. trigeminal B. facial C. oculomotor D. vagus E. glossopharyngeal 538. A car crash victim had his brachial artery ligated (tied) in the lower third of the upper arm. What arteries can restore blood flow to the forearm and hand after this procedure? A. thoracoacromial artery, posterior humeral circumflex artery B. muscular branches of brachial artery C. anterior and posterior humeral circumflex arteries D. subscapular artery, anterior interosseous artery E. deep artery of arm, ulnar collateral arteries 539. A 23 year-old patient complains of increased amount of saliva. Stimulation of what autonomic node caused increase in the amount of serous saliva? A. Ganglion oticum B. Ganglion pterygopalatinum C. Ganglion ciliare D Ganglion submandibulare E. Ganglion sublinguale 540. A patient has characteristic changes of gait, so-called "duck" gait: while walking he sways sideways. Adduction of legs is not possible. What nerve is damaged? A. tibial nerve B. femoral nerve C. superior gluteal nerve D. obturator nerve E. inferior gluteal nerve 541. A 62 year-old patient complains of sharp pain in the skin of the posterior side of his thigh down to his popliteal fossa. Damage of what of nerve could have caused this? A. inferior gluteal nerve B. femoral nerve C. lateral cutaneous nerve of thigh D. superior gluteal nerve E. posterior cutaneous nerve of thigh 542. The patient came to a clinic with a wound in the area of the neck. Damage of the nerve, located in front of anterior scalenus muscle was confirmed. What nerve is damaged? A. vagus B. diaphragmatic C. glossopharyngeal D. hypoglossus E. cervical sympathetic trunk 543. A 40 year-old male sustained a maxillofacial trauma; as the result, function of sublingual and submandibular glands on the left is impaired. Saliva from these glands is hardly excreted. Function of what nerve is danaged? A. XII pair B. VI pair C. X pair D. VII pair E. XI pair 544. A 30 year-old with cut wounds of forearms cannot extend his fingers. This is evidence of damage of: A. N. musculocutaneus B. N. ulnaris C. N. medianus D N. radialis E. N. cutaneus antebrachii medialis 545. A surgery was done to drain a deep abscess of a cheek, and vertical incision was made, that was complicated by paresis (dysfunction) of the muscles on the side of the operation. Branches of what nerve nerve were damaged? A. Mandibular B. Maxillary C. Facial D. Vagus E. Trigeminal 546. A patient had aortic arch aneurysm confirmed by an ultrasound examination. The patient’s voice had changed due to this condition. What nerve was damaged? A. recurrent laryngeal B. diaphragmatic C. superior laryngeal D. mandibular E. sublingual 547. A child is presumed to have mitral valve stenosis. What is the auscultation point of the valve? A. point of cardiac beat B. basis of xyphoid process C. second intercostal space near the right margin of sternum D. second intercostal space near the left margin of sternum E. fifth intercostal space near the right margin of sternum 548. A 56 year-old patient suffers from chronic hepatitis and has increased pressure in the system of vena porta. He is noted to have enlarged subcutaneous veins around his navel. What veins provide outflow of blood from the system of vena porta into this subcutaneous network? A. v.gastrica sinistra B. v.epigastrica superior C. vv.paraumbilicales D. v.epigastrica inferior E. v.lienalis 549. Entrapment of a sensory nerve may present clinical signs of paresthesia (numbness) extending from the medial aspect of the knee to the medial aspect of the foot. What is this nerve? A. saphenous nerve. B. tibial nerve. C. common peroneal nerve D. sural nerve. E. cutaneus surae medialis 550. After injection into the upper outer quadrant of gluteal area a patient started feeling pain while abducting his hip. What nerve was damaged as a complication of the injection? A. Superior gluteal nerve B. Internal obturator nerve C. Inferior gluteal nerve D. Pudendal nerve E. Sciatic nerve 551. A physician carries out auscultation of one of the heart valves in the area of the basis of xyphoid process of sternum. What valve is being auscultated? A. right atrioventricular B. of pulmonary trunk C. aortal D. left atrioventricular E. bicuspid 552. As a result of a trauma a patients had injured abducens nerve. What symptoms are observed? A. Paralysis of the medial rectus muscle of the eyeball B. Paralysis of the lateral rectus muscle of the eyeball C. Persistent mydriasis D. Disorder of accommodation E. tearing disorders 553. A physician carries out auscultation of one of the heart valves in the area of the basis of xyphoid process of sternum. What valve is being auscultated? A. left atrioventricular B. of pulmonary trunk C. aortal D. tricuspid E. bicuspid 554. A patient sustained head injuries, resulting in a hematoma in the middle cranial fossa on the left. His pupil is dilated on the affected side. What nerve is damaged? A. N.opticus B. N.abduceus C. N.oculomotorius D. N.trochlearis E. N.trigeminus 555. A patient has impaired function of masticatory muscles. What nerve is damaged? A. mandibular nerve B. lingual nerve C. auriculo-temporal nerve D. buccal nerve E. maxillary nerve 556. A patient has undergone a coronarography (radioligic examination of the coronary arteries with a contrast medium injection) for the purpose of ruling out ischaemic disease diagnosis. Left maincoronary artery bifurcates into the following branches: A. ramus interventricularis posterior and ramus descendens B. ramus interventricularis anterior and ramus ascendens C. ramus interventricularis anterior and ramus circumphlexus D. ramus interventricularis anterior and ramus interventricularis posterior E. ramus interventricularis anterior and ramus descendens 557. A 62 year-old man complains of the bloody discharge from rectum. Examination confirmed tumor that needs to be removed surgically. Branches of what artery supply rectum? A. a.mesenterica inferior and a.iliaca externa B. a.mesenterica inferior and a.iliaca interna C. a.mesenterica superior and a.mesenterica inferior D. a.iliaca interna and a.iliaca externa E. a.mesenterica superior and a.iliaca interna 558. The patient complains of pain in his gums, particularly those of the upper jaw. What nerve is affected? A. accesorius B. III branch of V pair C. I branch of V pair D. hypolossus E. II branch of V pair 559. Patient G. has an inflammatory purulent process of skin in the area of the first interdigital space of foot. What lymph nodes are regional for this area? A. Common iliac B. Deep inguinal C. Superficial inguinal D. Superficial iliac E. Internal iliac 560. Apatient has atrophy of the sternocleidomastoid and the upper edge of the trapezius muscles accompanied by difficulty turning head to the opposite direction. What nerve is affected? A. Hypoglossus B.Vagus C. Intercostal D. Brachial plexus E. Accessorius 561. A patient was admitted with diagnosis of “sinus tahicardia” (150 heart beats per minute). Sinoatrial node is located: A. in the wall of right atrium B. within interventricular septum C. in the wall of left atrium D. within interatrial septum E. in the wall of coronary sinus 562. Patient N., 52 years old, has complaints sensitivity loss on the right half of her face in the area of lower eyelids, back of the nose and upper lip. Indicate the branch of a nerve that is damaged. A. mandibular nerve of the trigeminal nerve B. greater petrosal nerve of facial nerve C. ophthalmic nerve of the trigeminal nerve D. maxillary nerve of the trigeminal nerve E. c horda tympani of the facial nerve 563. A patient admitted to a neurologic department had tongue deviation, atrophy of one half of the tongue, speech disorders, and difficulty swallowing. What nerve is damaged? A. hypoglossal B. lingual C. chorda tympani D. glossopharyngeal E. vagus 564. After traumatic damage of forearm, a patient cannot extend her hand at the wrist (wrist drop). What nerve is damaged? A.median nerve B. ulnar nerve C. radial nerve D. axillary nerve E. musculocutaneous nerve 565. A victim damaged his lower limb in an accident at the level of the upper third of tibia. Extension of the foot is not possible. What nerve is damaged? A. Tibial nerve B. Superficial peroneal nerve C. Deep peroneal nerve D. Femoral nerve E. Common peroneal nerve 566. A patient has no sensation in his pinky finger. What nerve is hurt? A. ulnar B. median C. radial D. musculocutaneous E. medial cutaneous of forearm 567. A patient has motor function of the tongue impairment. What nerve is damaged? A. Facial B. Vagus C. Glossopharyngeal D.Hypoglossus E. Accessorius 568. A patient has limited flexion of the elbow, tone of biceps is diminished, and skin sensitivity is absent on the anterior-lateral forearm. Function of what nerve is impaired? A. N. medianus B. N. axillaris C. N. musculocutaneus D. N. ulnaris E. N. Radialis 569. A patient has vasodilation on the anterior medial surface of tibia. Dalation of what vessel caused this condition? A. V. saphena magna B. А. tibialis anterior C. V. saphena parva D. А. tibialis posterior E. V. poplitea 570. A patient lost the ability to pronate his foot after a shin injury. Which nerve is damaged? A. Tibial nerve B. Deep peroneal nerve C. Superficial peroneal nerve D. Sciatic nerve E. Common peroneal nerve 571. Following a stabbed wound to the left palm, a 44 year-old patient had his tendons and superficial blood vessels damaged. After surgical removal of necrotic tissues blood circulation was restored. What vessels help restoration of the blood supply? A. Arcus palmaris profundus B. Arcus palmaris superficialis C. Aa. digitales palmares communes D. Aa. metacarpeae palmares E. Aa. metacarpeae dorsales 572. A patient is unable to abduct his right hand after a trauma. However, passive movements are not restricted. Deltoid muscle atrophy is noted upon examination. What nerve is damaged? A. median nerve B. ulnar nerve C. axillary nerve D. Radial nerve E. suprascapular nerve 573. A patient has lesion of the dorsal part of pons, his mastication is impaired. Nucleus of what nerve is affected? A. Motor nucleus of the facial nerve B. Motor nucleus of trigeminal nerve C. Principal trigeminal nucleus D. Nucleus of the hypoglossal nerve E. Ambiquus nucleus of the vagus nerve 574. A car crash victim has broken spinous processes and arch of XI thoracic vertebra. What segments of the spinal cord are damaged? A. Coccygeal B. Thoracic C. Sacral D. Lumbar E. Cervical 575. A patient underwent subtotal subfascial resection of the thyroid gland. Hoarseness of voice has been persistent in the postoperative period. What nerve was damaged during surgery? A. superior laryngeal nerve B. recurrent laryngeal nerve C. hypoglossal nerve D. lingual nerve E. mandibular nerve 576. A patient went to a neurologist who diagnosed a lesion of the facial nerve. Which of the following cranial nerves does not contain parasympathetic fibers? A. III B. XII C. VII D. X E. IX 577. A 65 year-old man complains of skin numbness on the medial surface of his shin and foot. What nerve is damaged? A. Superficial peroneal nerve B. Subcutaneous nerve C. Deep peroneal nerve D. Tibial nerve E. obturator nerve 578. A 40 year-old male patient has sustained a blow to his head and now has hearing impairment and paresis of facial muscles. The doctor diagnosed him with a hematoma of the cerebellopontine angle. What nerves are damaged in thus case? A. VII, VIII pairs of cranial nerves B. V, VI, pairs of cranial nerves C. VIII, IX pairs of cranial nerves D. IX, X, pairs of cranial nerves E. XI, XII pair of cranial nerves 579. The patient can not extend his shin, skin sensitivity on the anterior surface of the thigh is absent. Neurological examination confirmed nerve damage. What nerve is damaged? A. femoral nerve B. obturator nerve C. Sciatic nerve D. superior gluteal nerve E. inferior gluteal nerve 580. A 40 year-old patient presented with diminished skin sensitivity on the medial surface of his right leg compared to the healthy left side; the skin on the affected side is also cold to touch. What nerve is damaged? A. tibial nerve B. saphenous nerve C. common peroneal nerve D. sural nerve E. superficial branch of the peroneal nerve 581. A patient, that sustained a stabbed wound of the anterior aspect of the arm, presents with arterial bleeding. What artery was most likely damaged? A. a.subscapularis B. a.brachialis C. a.radialis D. a.axillaries E. aa. metacarpeae dorsales 582. A patient with a knife-cut wound of the anterior aspect of the arm accompanied by arterial bleeding presented to an emergency department. What artery was most likely damaged? A. a.brachialis B. a.subscapularis C. a.radialis D. a.axillaries E. aa. metacarpeae dorsales 583. A 40 year-old female patient complains of inability to extend her foot and toes, that causes difficulty walking. Upon examination her foot is hanging, facing slightly inward, her toes flexed ("horse's foot"), sensitivity is lost on the outer surface of the leg and dorsal surface of the foot. What nerve is affected? A. subcutaneous nerve B. tibial nerve C. sciatic nerve D. femoral nerve E. common peroneal nerve 584. A 36 year-old patient was diagnosed with loss of skin sensitivity and pain in the superciliary region of the forehead, cheekbones and chin on the right. Function of what nerve is damaged? A. N. trigeminus B. N. facialis C. N. oculomotorius D. N. trochlearis E. N. abducens 585. A surgeon operated a patient with an abscess of a deep lymph node in the axillary area. However, the surgery was complicated by loss of ability to flex the elbow and deteriorated skin sensitivity on anterior-lateral surface of the forearm. Injury of what brachial plexus nerve took place during surgery? A. N. ulnaris B. N. musculocutaneus C. N. medianus D N. radialis E. N. cutaneus antebrachii medialis 586. During medical check-up, a school student was diagnosed with harsh systolic murmur in the second intercostal space. Following additional diagnostic work-up persistence of arterial duct of Botallo was confirmed; the duct connects: A. right atrium with vena cava superior B. aorta with vena cava inferior C. aorta with vena cava superior D. right atrium with left atrium E. aorta with truncus pulmonalis 587. As a result of mandible dislocation, a patient has no sense of taste in the front of his tongue and constant tearing. What nerve is damaged? A. tympanic B. hypoglossus C. mandibular D. vagus E. facial 588. During examination of blood supply of foot, a physician feels pulsation of a large artery, that passes behind malleolus medialis in a separate fibrous canal. What artery is this? A. a.fibularis B. a.tibialis posterior C. a.tarsea medialis D. a.tibialis anterior E. a.tarsea lateralis 589. After suffering encephalitis, a patient has residual effects in the form of facial nerve damage on the left. Innervation of what muscles is impaired? A. subcutaneous neck muscles B. masticatory muscles C. facial muscles D. middle neck muscles E. deep neck muscles 590. While checking pupillary reflex in a patient, a doctor noted slow reaction to light on the left .Function of what nucleus is impaired? A. Nucleus of the of the trochlear nerve B. Accesory nucleus of the oculomotor nerve C. Nucleus of the abducent nerve D. Nuclei superior colliculi E. Nucleus of the of the oculomotor nerve 591. A patient was diagnosed with pneumonia. Lungs receive arterial blood supply through: A. pulmonary arteries B. branches of internal thoracic arteries C. bronchial branches of thoracic aorta D. branches of truncus brachiocephalicus E. branches of subclavian artery 592. A patient has diminished skin sensitivity on the lateral surface of the forearm. What nerve is damaged? A. medianus B. musculocutaneus C. radialis D. axilaris E. ulnaris 593. A patient complains of dizziness and hearing loss. What nerve is damaged? A. N.trigeminus B. N. hypoglossus C. vestibulocochlearis D. N.trochlearis E. vagus 594. A patient feels pain in the area of the root of the tongue, throat, tonsils, upper part of pharynx, ear, as well as lost of taste in the posterior third of the tongue. What nerve is damaged? A. Chorda tympani B. Vagus C. Lingual D. Glossopharyngeal E. Greater petrosal 595. A patient has decrease of tactile and taste sensation of mucosa in the posterior third of the back of the tongue. What nerve is damaged? A. hypoglossus B. the third branch of the trigeminal nerve C. facial D. glossopharyngeal E. the second branch of trigeminal nerve 596. During an accident, a 40 year-old man injured the lower part of his cervical spine, paralysis of the back, shoulder, and forearm muscles followed. What nerve was damaged? A. median nerve B. ulnar nerve C. radial nerve D. axillary nerve E. musculocutaneous nerve 597. A patient suffers from pain and edema in his lower extremities. Upon examination, edema and expansion of veins with varicosis are noted on the medial surface of the thigh. Pathology of what vein is present? A. v.basilica B.v.saphena parva C.v.cephalica D. v.saphena magna E. femoralis 598. A doctor diagnosed rupture of anterior cruciform ligament of the knee. What artery divides in cruciforms ligaments? These are: A. A. descendens genus B. A. superior medialis genus C. A. superior lateralis genus D. A. media genus E. A. inferior medialis genus 599. A 23 year-old patient complained to her dentist of increased amount of saliva. Stimulation of what autonomic ganglion causes increase in saliva formation? A. Ganglion oticum B. Ganglion pterygopalatinum C. Ganglion ciliare D Ganglion submandibulare E. Ganglion sublinguale 600. A has no sensation on the medial surface of the shoulder down to the elbow after an injury. What nerve is affected? A. ulnar nerve B. medial cutaneous nerve of forearm C. radial nerve D. medial cutaneous nerve of arm E. axillary nerve 601. As a result of cranial trauma and damage to the upper wall of the right eye socket a victim lost the ability to raise her right upper eyelid and to look up. What nerve is most likely damaged? A. N. trochlearis B. R. inferior n.oculumotorius C. R. superior n.oculumotorius D. N. abducens E. N. ophthalmicus 602. A patient with myocardial ischemia has considerable worsening of blood flow in the vein, that is located in sulcus interventricularis anterior of the heart. This vein is: A. vena obliqua atrii sinistri B.vena cordis media C.vena cordis parva D. vena cordis magna E.venae cordis anteriores 603. A patient with a lacerated wound in the area of popliteal fossa complains of diminished sensitivity on the posterior medial surface of his leg. What nerve is damaged? A. Tibial nerve B. Lateral cutaneous nerve of the shin C. Medial cutaneous nerve of the shin D. Peroneal nerve E Sural nerve 604. Patient A. was admitted to a neurological department with complaints of pain in the right side of his face. Upon examination decreased pain and touch sensitivity in this area, particularly in supra- and infraorbital points and chin was noted. What nerve is affected? A. Hypoglossal B. Facial C. Trigeminal D. Accessorius E. Cutaneous branches of brachial plexus 605. The surgeon, who operates abdominal part of esophagus, may accidentally injure a vessel. What vessel is this? A. a.gastroduodenalis B. a.gastrica dextra C.a.gastroepiploica sinistra D. a.gastrica sinistra E. a.gastroepiploica dextra 606. Patients with epidemic encephalitis often have uni- or bilateral ptosis (eyelid ptosis) and disturbance of accommodation. The pupils are dilated. What cranial nerves’ nuclei are affected? A. IV B. III C. V D. VI E. VII 607. A patient suffered acute myocardial infarction of the anterior wall and apex of the heart due to thrombosis of one of the coronary vessels. What artery is affected? A. circumflex branch branch of left coronary artery B. posterior interventricular branch of right coronary artery C. anterior interventricular branch of left coronary artery D. marginal branch of left coronary artery E. marginal branch of right coronary artery 608. A patient reports pain, swelling and redness in the anterolateral thigh and the tip of his big toe. What are the lymph nodes that would react to the inflammation? A. Common iliac B. Deep inguinal C. Internal iliac D.Superficial iliac E. Superficial inguinal 609. A patient had varices and thrombophlebitis on the medial surface of his leg. What vein is affected? A. А. tibialis anterior B. V. saphena magna C. V. saphena parva D. А. tibialis posterior E. V. poplitea 610. A 36 year-old patient is suffering convulsive contractions of the diaphragm after a surgery. What nerve block should be done to treat this complication ? A. N. frenicus B. N. vagus C. N. splanchnicus major D. N. accessorius E. Tr. Sympathicus 611. Following a trauma of the anterior upper third of forearm, a patient has impaired pronation, weakened palmar flexion and skin sensitivity changes in his 1-3 fingers. What nerve is damaged? A. N. musculocutaneus B. N. medianus C. N. ulnaris D. N. cutaneus antebrachii medialis E. N. radialis 612. As a result of trauma, a patients injured abducens nerve. What symptoms are observed? A. Paralysis of the medial rectus muscle of the eyeball. B. Paralysis of the lateral rectus muscle of the eyeball. C. Persistent mydriasis. D. Disorder accommodation. E. Violation of tearing. 613. A 45 year-old patient has pale skin and absence of pulse on arteria dorsalis pedis and arteria tibialis posterior on the right. His pulse on the femoral artery is intact. What artery is affected? A. tibial anterior B. external iliac C.deep femoral D.fibular E. popliteal 614. A patient has a tumor of the eye socket tissues behind the eyeball. His accomodation and pupil constriction are impaired. What structure is damaged? A. Ganglion ciliare. B. N.nasociliaris. C. N. trochlearis. D. N.lacrimalis. E. N.opticus. 615. A 29 year-old man with a stabbed wound of the neck presents with bleeding. During the initial cleaning of the wound a surgeon noted damage to a vessel situated along the lateral edge of the sternocleidomastoid muscle. What is this vessel? A. A.carotis interna. B. V.jugularis anterior. C. A.carotis externa. D. V.jugularis externa. E. V.jugularis interna. 616. Angiography revealed stenosis of a vessel located in the left coronary sulcus of the heart. Name this vessel: A. Ramus interventricularis posterior. B. V.cordis parva. C. Ramus interventricularis anterior. D. A.coronaria dextra. E. Ramus circumflexus. 617. While examining foot blood supply, a doctor checks pulsation of a large artery running in a separate fibrous canal in the front of articulatio talocruralis between the tendons of extensor hallucis longus and extensor digitorum. What artery is it? A. a.fibularis. B. a. tibialis anterior. C. a.tarsea lateralis. D. a.tarsea medialis. E. a.dorsalis pedis. 618. During appendectomy a patient had arteria appendicularis ligated. This vessel branches from the following artery: A.a ileocolica. B.a.sigmoidea. C. a.mesenterica inferior. D. a.colica dextra. E.a.colica media 619. A child has a wound located posteriorly to the mastoid process. Bright red blood flows from the wound. What artery branches are damaged? A. a.carotis interna. B. a.carotis interna. C. a. occipitalis. D. a. maxillaris. E. a. temporalis superior. 620. A patient with suspected necrosis of the upper abdominal cavity organs was admitted to a surgical department. This condition is associated with acute circulatory disturbance in the following vessels: A. a.mesenterica superior. B. a.iliaca communis. C. Truncus coeliacus. D.a.renalis. E.a.mesenterica inferior 621. A man arrived to a traumatology department with a trauma of the right shoulder. Examination revealed humeral shaft fracture in the medial third on the right with fragments displacement; the patient can't extend his fingers on the affected side. What nerve is damaged? A) Musculocutaneus. B) Radial C) Ulnar D) Axillary E) Median

474. c 475. c 476. d 477. b 478. b 479. e 480. e 481. e 482. c 483. d 484. d 485. e 486. d 487. b 488. a 489. a 490. b 491. e 492. e 493. e 494. d 495. a 496. a 497. d 498. a 499. e 500. b 501. b 502. a 503. e 504. c 505. a 506. e 507. a 508. c 509. e 510. d 511. e 512. c 513. e 514. a 515. a 516. c 517. d 518. a 519. b 520. e 521. d 522. a 523. d 524. e 525. a 526. a 527. d 528. d 529. e 530. b 531. d 532. b 533. a 534. e 535. c 536. b 537. a 538. e 539. a 540. c 541. e 542. b 543. d 544. d 545. c 546. a 547. a 548. c 549. a 550. a 551. a 552. b 553. d 554. c 555. a 556. c 557. b 558. e 559. c 560. e 561. a 562. d 563. a 564. c 565. c 566. a 567. d 568. c 569. a 570. c 571. a 572. c 573. b 574. d 575. b 576. b 577. b 578. a 579. a 580. b 581. b 582. a 583. e 584. a 585. b 586. e 587. e 588. b 589. c 590. b 591. c 592. b 593. c 594. d 595. d 596. c 597. d 598. d 599. a 600. d 601. c 602. d 603. c 604. c 605. d 606. b 607. c 608. e 609. b 610. a 611. b 612. b 613. e 614. b 615. d 616. e 617. d 618. a 619. c 620. c 621. b